Rocksolid Light

Welcome to novaBBS (click a section below)

mail  files  register  newsreader  groups  login

Message-ID:  

"We will bury you." -- Nikita Kruschev


tech / sci.electronics.design / Re: Faraday bags and other scarey things

SubjectAuthor
* Re: Faraday bags and other scarey thingsJoe Gwinn
`* Re: Faraday bags and other scarey thingsJohn Larkin
 +* Re: Faraday bags and other scarey thingsJoe Gwinn
 |+* Re: Faraday bags and other scarey thingsRicky
 ||`* Re: Faraday bags and other scarey thingsJoe Gwinn
 || `* Re: Faraday bags and other scarey thingsRicky
 ||  `- Re: Faraday bags and other scarey thingsJoe Gwinn
 |`* Re: Faraday bags and other scarey thingsJohn Larkin
 | +* Re: Faraday bags and other scarey thingsJoe Gwinn
 | |`- Re: Faraday bags and other scarey thingsJohn Larkin
 | +* Re: Faraday bags and other scarey thingsPhil Hobbs
 | |`* Re: Faraday bags and other scarey thingsJohn Larkin
 | | `* Re: Faraday bags and other scarey thingsJoe Gwinn
 | |  +* Re: Faraday bags and other scarey thingsPhil Hobbs
 | |  |`* Re: Faraday bags and other scarey thingsJohn Larkin
 | |  | `* Re: Faraday bags and other scarey thingsPhil Hobbs
 | |  |  +- Re: Faraday bags and other scarey thingsRicky
 | |  |  `* Re: Faraday bags and other scarey thingsJohn Larkin
 | |  |   `- Re: Faraday bags and other scarey thingsa a
 | |  `- Re: Faraday bags and other scarey thingsJohn Larkin
 | `* Re: Faraday bags and other scarey thingsmarty
 |  `- Re: Faraday bags and other scarey thingsJohn Larkin
 `* Re: Faraday bags and other scarey thingswhit3rd
  `* Re: Faraday bags and other scarey thingsJohn Larkin
   `* Re: Faraday bags and other scarey thingsPhil Hobbs
    +* Re: Faraday bags and other scarey thingswhit3rd
    |`* Re: Faraday bags and other scarey thingsPhil Hobbs
    | `- Re: Faraday bags and other scarey thingswhit3rd
    `- Re: Faraday bags and other scarey thingsJohn Larkin

Pages:12
Re: Faraday bags and other scarey things

<0cmpqhhgqe6jv3kltna5broqnooj1rbd4i@4ax.com>

  copy mid

https://www.novabbs.com/tech/article-flat.php?id=113191&group=sci.electronics.design#113191

  copy link   Newsgroups: sci.electronics.design
Path: i2pn2.org!i2pn.org!usenet.blueworldhosting.com!feed1.usenet.blueworldhosting.com!peer03.iad!feed-me.highwinds-media.com!news.highwinds-media.com!Xl.tags.giganews.com!local-1.nntp.ord.giganews.com!news.giganews.com.POSTED!not-for-mail
NNTP-Posting-Date: Thu, 29 Dec 2022 00:09:11 +0000
From: joegw...@comcast.net (Joe Gwinn)
Newsgroups: sci.electronics.design
Subject: Re: Faraday bags and other scarey things
Date: Wed, 28 Dec 2022 19:09:10 -0500
Message-ID: <0cmpqhhgqe6jv3kltna5broqnooj1rbd4i@4ax.com>
References: <0jbpqh9glc5nnc353uanjt33165o4reqao@4ax.com> <toieom$2r24$1@dont-email.me> <43kpqhptg3bu556300bqemp473to7kvml2@4ax.com>
User-Agent: ForteAgent/8.00.32.1272
MIME-Version: 1.0
Content-Type: text/plain; charset=us-ascii
Content-Transfer-Encoding: 7bit
Lines: 77
X-Usenet-Provider: http://www.giganews.com
X-Trace: sv3-ro1EDO1PeTr+hjl6dTsdBf2Mo2JLfBY4/5ZSQnqX8X9Lnsw5zUnPnP4oA8sgi2XFiDttzsvAp8NjiaY!e0dFRkyoQUvoNzIsM01z0zP04xqR9yyFwMHyEP3FTsLsifcJNjR5SIf6xzKYmOkoH/pTr2w=
X-Complaints-To: abuse@giganews.com
X-DMCA-Notifications: http://www.giganews.com/info/dmca.html
X-Abuse-and-DMCA-Info: Please be sure to forward a copy of ALL headers
X-Abuse-and-DMCA-Info: Otherwise we will be unable to process your complaint properly
X-Postfilter: 1.3.40
X-Received-Bytes: 4935
 by: Joe Gwinn - Thu, 29 Dec 2022 00:09 UTC

On Wed, 28 Dec 2022 15:32:07 -0800, John Larkin
<jlarkin@highland_atwork_technology.com> wrote:

>On Wed, 28 Dec 2022 22:08:22 -0000 (UTC), Sergey Kubushyn
><ksi@koi8.net> wrote:
>
>>In sci.electronics.repair micky <NONONOmisc07@fmguy.com> wrote:
>>>
>>> https://www.moneyshake.com/car-finance-guides/road-safety/how-to-protect-keyless-entry-cars
>>> Unlike standard cars where access without raising the alarm requires the
>>> key, keyless entry models can be ?tricked? into thinking a key is
>>> present through a transmitter and receiver device.
>>> If your car?s fob isn?t stored safely, then it can be very easy for a
>>> pair of opportune thieves to pick up the key?s radio frequency signal,
>>> without having to break into your house. Luckily, however, there are
>>> many ways in which you can make it extremely difficult for them to do
>>> this and protect your motor.
>>>
>>> ---- Except, how isthere any signal to pick up unless you are in your
>>> home pressing the Unlock button on the fob at the same time the thief
>>> drives by? I myself dont' bother pressing fob buttons once I've left
>>> the car, until I want to drive away the next day.
>>>
>>> https://www.wavy.com/news/local-news/norfolk/relay-devices-being-used-to-steal-locked-cars/
>>
>>When I'm close to my Lexus LS460L with keyfob in my pocket it greets me with
>>turning on the lights under rearview mirrors.
>>
>>Then, I pull the door handle and it opens the door and unfolds the rearview
>>mirrors. Then, I step on the brake pedal and push the car's (not keyfob)
>>START button and it starts up.
>>
>>Also, when I'm touching a button on the trunk with my keyfob in my pocket it
>>opens the trunk.
>>
>>None of that happens when I have that keyfob in a copper mesh Faraday
>>bag/protector no matter how close I am to my car.
>>
>>I don't remember pushing ANY keyfob button EVER for as long as I own that
>>car. I'm locking it when leaving by pressing a button on any door handle. It
>>locks all doors, folds the mirrors and activates security system when a
>>button on any door handle is pressed when keyfob is nearby and not in
>>Faraday bag. Won't do it if the keyfob is inside the car.
>>
>>And this is true for ALL relatively modern cars with that START button. They
>>are ALL prone to stealing by an amplifier/receiver if keyfob is not put in a
>>Faraday bag. There was a video from security camera on our local Nextdoor
>>showing how a top-of-the-line Mercedes-Benz was stolen from our neighbor's
>>driveway in less than a minute, from a car stopping by and a perp going out
>>of that car to the moment he drove away in that Mercedes.
>>
>>Please, don't spread that BS about no signal unless a keyfob button is
>>pressed. It is simply false.
>>
>>---
>>******************************************************************
>>* KSI@home KOI8 Net < > The impossible we do immediately. *
>>* Las Vegas NV, USA < > Miracles require 24-hour notice. *
>>******************************************************************
>
>When we get home, our keys go in a Danish Butter Cookie can. Those are
>pretty good shields.
>
>https://www.dropbox.com/s/30c6aesh4tzjg56/Z496_Can.jpg?raw=1
>
>https://www.dropbox.com/s/cuxn5r7yzenb8m2/Z496_Out.jpg?raw=1

I bet keeping the fobs in an open top brass bowl would also work, by
being a shorted turn close to the fob's antenna loop.

Back when we worried about the RFID chips un US passports being read
from a distance, I would but a piece of 0.003" brass foil in the
passports book. If I forgot to remove the brass sheet, the US
Passport Control folk could not read the RFID chip. They never gave
any grief after I removed the sheet and it scanned OK.

Joe Gwinn

Re: Faraday bags and other scarey things

<sk2uqhtep8iojr9v1r45dobfp928m0cpmr@4ax.com>

  copy mid

https://www.novabbs.com/tech/article-flat.php?id=113312&group=sci.electronics.design#113312

  copy link   Newsgroups: sci.electronics.design
Path: i2pn2.org!i2pn.org!usenet.blueworldhosting.com!feed1.usenet.blueworldhosting.com!peer03.iad!feed-me.highwinds-media.com!news.highwinds-media.com!feeder.usenetexpress.com!tr1.iad1.usenetexpress.com!69.80.99.26.MISMATCH!Xl.tags.giganews.com!local-2.nntp.ord.giganews.com!nntp.supernews.com!news.supernews.com.POSTED!not-for-mail
NNTP-Posting-Date: Fri, 30 Dec 2022 15:59:21 +0000
From: jlar...@highlandSNIPMEtechnology.com (John Larkin)
Newsgroups: sci.electronics.design
Subject: Re: Faraday bags and other scarey things
Date: Fri, 30 Dec 2022 07:59:23 -0800
Organization: Highland Tech
Reply-To: xx@yy.com
Message-ID: <sk2uqhtep8iojr9v1r45dobfp928m0cpmr@4ax.com>
References: <0jbpqh9glc5nnc353uanjt33165o4reqao@4ax.com> <toieom$2r24$1@dont-email.me> <43kpqhptg3bu556300bqemp473to7kvml2@4ax.com> <0cmpqhhgqe6jv3kltna5broqnooj1rbd4i@4ax.com>
X-Newsreader: Forte Agent 3.1/32.783
MIME-Version: 1.0
Content-Type: text/plain; charset=us-ascii
Content-Transfer-Encoding: 7bit
Lines: 77
X-Trace: sv3-DYVS0SezpVZRZThzSUwFXBU3OZdQm7l2UHcN67sPagoNHWCgvYhQ5iykFqIEN3S7ewjLA1Mcy5p6hc9!Pm/jJ1s5sYEoWWGMcomMduc36Pj3qY/Wu6lcumslPVRd9pXa9G+JM6STmxvcKEOKeKjXnOpiuBBr!NCG1qQ==
X-Complaints-To: www.supernews.com/docs/abuse.html
X-DMCA-Complaints-To: www.supernews.com/docs/dmca.html
X-Abuse-and-DMCA-Info: Please be sure to forward a copy of ALL headers
X-Abuse-and-DMCA-Info: Otherwise we will be unable to process your complaint properly
X-Postfilter: 1.3.40
X-Received-Bytes: 4998
 by: John Larkin - Fri, 30 Dec 2022 15:59 UTC

On Wed, 28 Dec 2022 19:09:10 -0500, Joe Gwinn <joegwinn@comcast.net>
wrote:

>On Wed, 28 Dec 2022 15:32:07 -0800, John Larkin
><jlarkin@highland_atwork_technology.com> wrote:
>
>>On Wed, 28 Dec 2022 22:08:22 -0000 (UTC), Sergey Kubushyn
>><ksi@koi8.net> wrote:
>>
>>>In sci.electronics.repair micky <NONONOmisc07@fmguy.com> wrote:
>>>>
>>>> https://www.moneyshake.com/car-finance-guides/road-safety/how-to-protect-keyless-entry-cars
>>>> Unlike standard cars where access without raising the alarm requires the
>>>> key, keyless entry models can be ?tricked? into thinking a key is
>>>> present through a transmitter and receiver device.
>>>> If your car?s fob isn?t stored safely, then it can be very easy for a
>>>> pair of opportune thieves to pick up the key?s radio frequency signal,
>>>> without having to break into your house. Luckily, however, there are
>>>> many ways in which you can make it extremely difficult for them to do
>>>> this and protect your motor.
>>>>
>>>> ---- Except, how isthere any signal to pick up unless you are in your
>>>> home pressing the Unlock button on the fob at the same time the thief
>>>> drives by? I myself dont' bother pressing fob buttons once I've left
>>>> the car, until I want to drive away the next day.
>>>>
>>>> https://www.wavy.com/news/local-news/norfolk/relay-devices-being-used-to-steal-locked-cars/
>>>
>>>When I'm close to my Lexus LS460L with keyfob in my pocket it greets me with
>>>turning on the lights under rearview mirrors.
>>>
>>>Then, I pull the door handle and it opens the door and unfolds the rearview
>>>mirrors. Then, I step on the brake pedal and push the car's (not keyfob)
>>>START button and it starts up.
>>>
>>>Also, when I'm touching a button on the trunk with my keyfob in my pocket it
>>>opens the trunk.
>>>
>>>None of that happens when I have that keyfob in a copper mesh Faraday
>>>bag/protector no matter how close I am to my car.
>>>
>>>I don't remember pushing ANY keyfob button EVER for as long as I own that
>>>car. I'm locking it when leaving by pressing a button on any door handle. It
>>>locks all doors, folds the mirrors and activates security system when a
>>>button on any door handle is pressed when keyfob is nearby and not in
>>>Faraday bag. Won't do it if the keyfob is inside the car.
>>>
>>>And this is true for ALL relatively modern cars with that START button. They
>>>are ALL prone to stealing by an amplifier/receiver if keyfob is not put in a
>>>Faraday bag. There was a video from security camera on our local Nextdoor
>>>showing how a top-of-the-line Mercedes-Benz was stolen from our neighbor's
>>>driveway in less than a minute, from a car stopping by and a perp going out
>>>of that car to the moment he drove away in that Mercedes.
>>>
>>>Please, don't spread that BS about no signal unless a keyfob button is
>>>pressed. It is simply false.
>>>
>>>---
>>>******************************************************************
>>>* KSI@home KOI8 Net < > The impossible we do immediately. *
>>>* Las Vegas NV, USA < > Miracles require 24-hour notice. *
>>>******************************************************************
>>
>>When we get home, our keys go in a Danish Butter Cookie can. Those are
>>pretty good shields.
>>
>>https://www.dropbox.com/s/30c6aesh4tzjg56/Z496_Can.jpg?raw=1
>>
>>https://www.dropbox.com/s/cuxn5r7yzenb8m2/Z496_Out.jpg?raw=1
>
>I bet keeping the fobs in an open top brass bowl would also work, by
>being a shorted turn close to the fob's antenna loop.

I doubt it.

Put the fob in the bowl and walk towards the car.

Re: Faraday bags and other scarey things

<cucuqh19jgbdvclqam5m2le2227ve4gcr6@4ax.com>

  copy mid

https://www.novabbs.com/tech/article-flat.php?id=113339&group=sci.electronics.design#113339

  copy link   Newsgroups: sci.electronics.design
Path: i2pn2.org!i2pn.org!usenet.blueworldhosting.com!feed1.usenet.blueworldhosting.com!peer03.iad!feed-me.highwinds-media.com!news.highwinds-media.com!Xl.tags.giganews.com!local-1.nntp.ord.giganews.com!news.giganews.com.POSTED!not-for-mail
NNTP-Posting-Date: Fri, 30 Dec 2022 19:22:09 +0000
From: joegw...@comcast.net (Joe Gwinn)
Newsgroups: sci.electronics.design
Subject: Re: Faraday bags and other scarey things
Date: Fri, 30 Dec 2022 14:22:08 -0500
Message-ID: <cucuqh19jgbdvclqam5m2le2227ve4gcr6@4ax.com>
References: <0jbpqh9glc5nnc353uanjt33165o4reqao@4ax.com> <toieom$2r24$1@dont-email.me> <43kpqhptg3bu556300bqemp473to7kvml2@4ax.com> <0cmpqhhgqe6jv3kltna5broqnooj1rbd4i@4ax.com> <sk2uqhtep8iojr9v1r45dobfp928m0cpmr@4ax.com>
User-Agent: ForteAgent/8.00.32.1272
MIME-Version: 1.0
Content-Type: text/plain; charset=us-ascii
Content-Transfer-Encoding: 7bit
Lines: 109
X-Usenet-Provider: http://www.giganews.com
X-Trace: sv3-k04dHHrO48/GSU8bWfbxpHgQ/Qcc12OO+u3uNmVxYrDNVo3WOyNULm7tAQGnL17/yXZGYwMAK3cfftV!LtjCFLwaynvvWsT6oxsavZ6RBbux0WYlh9dFi1U+nxzA+Nw4nyoXlrEO7EWJOMyVjAllSLo=
X-Complaints-To: abuse@giganews.com
X-DMCA-Notifications: http://www.giganews.com/info/dmca.html
X-Abuse-and-DMCA-Info: Please be sure to forward a copy of ALL headers
X-Abuse-and-DMCA-Info: Otherwise we will be unable to process your complaint properly
X-Postfilter: 1.3.40
X-Received-Bytes: 6382
 by: Joe Gwinn - Fri, 30 Dec 2022 19:22 UTC

On Fri, 30 Dec 2022 07:59:23 -0800, John Larkin
<jlarkin@highlandSNIPMEtechnology.com> wrote:

>On Wed, 28 Dec 2022 19:09:10 -0500, Joe Gwinn <joegwinn@comcast.net>
>wrote:
>
>>On Wed, 28 Dec 2022 15:32:07 -0800, John Larkin
>><jlarkin@highland_atwork_technology.com> wrote:
>>
>>>On Wed, 28 Dec 2022 22:08:22 -0000 (UTC), Sergey Kubushyn
>>><ksi@koi8.net> wrote:
>>>
>>>>In sci.electronics.repair micky <NONONOmisc07@fmguy.com> wrote:
>>>>>
>>>>> https://www.moneyshake.com/car-finance-guides/road-safety/how-to-protect-keyless-entry-cars
>>>>> Unlike standard cars where access without raising the alarm requires the
>>>>> key, keyless entry models can be ?tricked? into thinking a key is
>>>>> present through a transmitter and receiver device.
>>>>> If your car?s fob isn?t stored safely, then it can be very easy for a
>>>>> pair of opportune thieves to pick up the key?s radio frequency signal,
>>>>> without having to break into your house. Luckily, however, there are
>>>>> many ways in which you can make it extremely difficult for them to do
>>>>> this and protect your motor.
>>>>>
>>>>> ---- Except, how isthere any signal to pick up unless you are in your
>>>>> home pressing the Unlock button on the fob at the same time the thief
>>>>> drives by? I myself dont' bother pressing fob buttons once I've left
>>>>> the car, until I want to drive away the next day.
>>>>>
>>>>> https://www.wavy.com/news/local-news/norfolk/relay-devices-being-used-to-steal-locked-cars/
>>>>
>>>>When I'm close to my Lexus LS460L with keyfob in my pocket it greets me with
>>>>turning on the lights under rearview mirrors.
>>>>
>>>>Then, I pull the door handle and it opens the door and unfolds the rearview
>>>>mirrors. Then, I step on the brake pedal and push the car's (not keyfob)
>>>>START button and it starts up.
>>>>
>>>>Also, when I'm touching a button on the trunk with my keyfob in my pocket it
>>>>opens the trunk.
>>>>
>>>>None of that happens when I have that keyfob in a copper mesh Faraday
>>>>bag/protector no matter how close I am to my car.
>>>>
>>>>I don't remember pushing ANY keyfob button EVER for as long as I own that
>>>>car. I'm locking it when leaving by pressing a button on any door handle. It
>>>>locks all doors, folds the mirrors and activates security system when a
>>>>button on any door handle is pressed when keyfob is nearby and not in
>>>>Faraday bag. Won't do it if the keyfob is inside the car.
>>>>
>>>>And this is true for ALL relatively modern cars with that START button. They
>>>>are ALL prone to stealing by an amplifier/receiver if keyfob is not put in a
>>>>Faraday bag. There was a video from security camera on our local Nextdoor
>>>>showing how a top-of-the-line Mercedes-Benz was stolen from our neighbor's
>>>>driveway in less than a minute, from a car stopping by and a perp going out
>>>>of that car to the moment he drove away in that Mercedes.
>>>>
>>>>Please, don't spread that BS about no signal unless a keyfob button is
>>>>pressed. It is simply false.
>>>>
>>>>---
>>>>******************************************************************
>>>>* KSI@home KOI8 Net < > The impossible we do immediately. *
>>>>* Las Vegas NV, USA < > Miracles require 24-hour notice. *
>>>>******************************************************************
>>>
>>>When we get home, our keys go in a Danish Butter Cookie can. Those are
>>>pretty good shields.
>>>
>>>https://www.dropbox.com/s/30c6aesh4tzjg56/Z496_Can.jpg?raw=1
>>>
>>>https://www.dropbox.com/s/cuxn5r7yzenb8m2/Z496_Out.jpg?raw=1
>>
>>I bet keeping the fobs in an open top brass bowl would also work, by
>>being a shorted turn close to the fob's antenna loop.
>
>I doubt it.
>
>Put the fob in the bowl and walk towards the car.

Good idea, and now it's warm enough.

The car being tested is a 2018 Honda CR-V.

The bowl did reduce range if one pushed the lock/unlock buttons using
a 9" wooden stick (versus one's finger), but not enough to solve the
problem I'd guess.

Bowl is made of spun 0.0355" thick brass, open top diameter 7.4", and
depth 3.3". Base, also same kind of brass, is soft soldered to the
bowl, looks to be done by hand with a big soldering iron (not a
torch). Quite old, has no markings whatsoever, likely US made.

Also tried a 12" square sheet of 0.062" thick brass - same result.

A cover ought to help, but this will take some fiddling to be able to
push the buttons through a solid metal sheet. Although I do have some
copper insect screening that could be pressed into service. One can
also make a bag from this screening fabric.

Longer term, I think that the entire class of man-in-the-middle
attacks on key fobs will be solved by moving to some kind of
ultra-wideband signal, where one can measure round-trip time to with
picosecond precision, using a crypto-key sequence that cannot be
deduced from received signals fast enough to matter, thus allowing
too-distant fobs to be ignored with sufficiently low leakage to make
this kind of attack unprofitable.

Joe Gwinn

Re: Faraday bags and other scarey things

<27a4a011-c5c5-4a64-9ed1-2ae880b94602n@googlegroups.com>

  copy mid

https://www.novabbs.com/tech/article-flat.php?id=113344&group=sci.electronics.design#113344

  copy link   Newsgroups: sci.electronics.design
X-Received: by 2002:a05:6214:5441:b0:531:832c:c227 with SMTP id kz1-20020a056214544100b00531832cc227mr959187qvb.22.1672430182552;
Fri, 30 Dec 2022 11:56:22 -0800 (PST)
X-Received: by 2002:ac8:7cac:0:b0:3a9:8901:6026 with SMTP id
z12-20020ac87cac000000b003a989016026mr1652220qtv.29.1672430182323; Fri, 30
Dec 2022 11:56:22 -0800 (PST)
Path: i2pn2.org!i2pn.org!usenet.blueworldhosting.com!feed1.usenet.blueworldhosting.com!peer02.iad!feed-me.highwinds-media.com!news.highwinds-media.com!news-out.google.com!nntp.google.com!postnews.google.com!google-groups.googlegroups.com!not-for-mail
Newsgroups: sci.electronics.design
Date: Fri, 30 Dec 2022 11:56:22 -0800 (PST)
In-Reply-To: <cucuqh19jgbdvclqam5m2le2227ve4gcr6@4ax.com>
Injection-Info: google-groups.googlegroups.com; posting-host=65.207.89.54; posting-account=I-_H_woAAAA9zzro6crtEpUAyIvzd19b
NNTP-Posting-Host: 65.207.89.54
References: <0jbpqh9glc5nnc353uanjt33165o4reqao@4ax.com> <toieom$2r24$1@dont-email.me>
<43kpqhptg3bu556300bqemp473to7kvml2@4ax.com> <0cmpqhhgqe6jv3kltna5broqnooj1rbd4i@4ax.com>
<sk2uqhtep8iojr9v1r45dobfp928m0cpmr@4ax.com> <cucuqh19jgbdvclqam5m2le2227ve4gcr6@4ax.com>
User-Agent: G2/1.0
MIME-Version: 1.0
Message-ID: <27a4a011-c5c5-4a64-9ed1-2ae880b94602n@googlegroups.com>
Subject: Re: Faraday bags and other scarey things
From: gnuarm.d...@gmail.com (Ricky)
Injection-Date: Fri, 30 Dec 2022 19:56:22 +0000
Content-Type: text/plain; charset="UTF-8"
Content-Transfer-Encoding: quoted-printable
X-Received-Bytes: 7927
 by: Ricky - Fri, 30 Dec 2022 19:56 UTC

On Friday, December 30, 2022 at 2:22:21 PM UTC-5, Joe Gwinn wrote:
> On Fri, 30 Dec 2022 07:59:23 -0800, John Larkin
> <jla...@highlandSNIPMEtechnology.com> wrote:
>
> >On Wed, 28 Dec 2022 19:09:10 -0500, Joe Gwinn <joeg...@comcast.net>
> >wrote:
> >
> >>On Wed, 28 Dec 2022 15:32:07 -0800, John Larkin
> >><jlarkin@highland_atwork_technology.com> wrote:
> >>
> >>>On Wed, 28 Dec 2022 22:08:22 -0000 (UTC), Sergey Kubushyn
> >>><k...@koi8.net> wrote:
> >>>
> >>>>In sci.electronics.repair micky <NONONO...@fmguy.com> wrote:
> >>>>>
> >>>>> https://www.moneyshake.com/car-finance-guides/road-safety/how-to-protect-keyless-entry-cars
> >>>>> Unlike standard cars where access without raising the alarm requires the
> >>>>> key, keyless entry models can be ?tricked? into thinking a key is
> >>>>> present through a transmitter and receiver device.
> >>>>> If your car?s fob isn?t stored safely, then it can be very easy for a
> >>>>> pair of opportune thieves to pick up the key?s radio frequency signal,
> >>>>> without having to break into your house. Luckily, however, there are
> >>>>> many ways in which you can make it extremely difficult for them to do
> >>>>> this and protect your motor.
> >>>>>
> >>>>> ---- Except, how isthere any signal to pick up unless you are in your
> >>>>> home pressing the Unlock button on the fob at the same time the thief
> >>>>> drives by? I myself dont' bother pressing fob buttons once I've left
> >>>>> the car, until I want to drive away the next day.
> >>>>>
> >>>>> https://www.wavy.com/news/local-news/norfolk/relay-devices-being-used-to-steal-locked-cars/
> >>>>
> >>>>When I'm close to my Lexus LS460L with keyfob in my pocket it greets me with
> >>>>turning on the lights under rearview mirrors.
> >>>>
> >>>>Then, I pull the door handle and it opens the door and unfolds the rearview
> >>>>mirrors. Then, I step on the brake pedal and push the car's (not keyfob)
> >>>>START button and it starts up.
> >>>>
> >>>>Also, when I'm touching a button on the trunk with my keyfob in my pocket it
> >>>>opens the trunk.
> >>>>
> >>>>None of that happens when I have that keyfob in a copper mesh Faraday
> >>>>bag/protector no matter how close I am to my car.
> >>>>
> >>>>I don't remember pushing ANY keyfob button EVER for as long as I own that
> >>>>car. I'm locking it when leaving by pressing a button on any door handle. It
> >>>>locks all doors, folds the mirrors and activates security system when a
> >>>>button on any door handle is pressed when keyfob is nearby and not in
> >>>>Faraday bag. Won't do it if the keyfob is inside the car.
> >>>>
> >>>>And this is true for ALL relatively modern cars with that START button. They
> >>>>are ALL prone to stealing by an amplifier/receiver if keyfob is not put in a
> >>>>Faraday bag. There was a video from security camera on our local Nextdoor
> >>>>showing how a top-of-the-line Mercedes-Benz was stolen from our neighbor's
> >>>>driveway in less than a minute, from a car stopping by and a perp going out
> >>>>of that car to the moment he drove away in that Mercedes.
> >>>>
> >>>>Please, don't spread that BS about no signal unless a keyfob button is
> >>>>pressed. It is simply false.
> >>>>
> >>>>---
> >>>>******************************************************************
> >>>>* KSI@home KOI8 Net < > The impossible we do immediately. *
> >>>>* Las Vegas NV, USA < > Miracles require 24-hour notice. *
> >>>>******************************************************************
> >>>
> >>>When we get home, our keys go in a Danish Butter Cookie can. Those are
> >>>pretty good shields.
> >>>
> >>>https://www.dropbox.com/s/30c6aesh4tzjg56/Z496_Can.jpg?raw=1
> >>>
> >>>https://www.dropbox.com/s/cuxn5r7yzenb8m2/Z496_Out.jpg?raw=1
> >>
> >>I bet keeping the fobs in an open top brass bowl would also work, by
> >>being a shorted turn close to the fob's antenna loop.
> >
> >I doubt it.
> >
> >Put the fob in the bowl and walk towards the car.
> Good idea, and now it's warm enough.
>
> The car being tested is a 2018 Honda CR-V.
>
> The bowl did reduce range if one pushed the lock/unlock buttons using
> a 9" wooden stick (versus one's finger), but not enough to solve the
> problem I'd guess.
>
> Bowl is made of spun 0.0355" thick brass, open top diameter 7.4", and
> depth 3.3". Base, also same kind of brass, is soft soldered to the
> bowl, looks to be done by hand with a big soldering iron (not a
> torch). Quite old, has no markings whatsoever, likely US made.
>
> Also tried a 12" square sheet of 0.062" thick brass - same result.
>
> A cover ought to help, but this will take some fiddling to be able to
> push the buttons through a solid metal sheet. Although I do have some
> copper insect screening that could be pressed into service. One can
> also make a bag from this screening fabric.
>
> Longer term, I think that the entire class of man-in-the-middle
> attacks on key fobs will be solved by moving to some kind of
> ultra-wideband signal, where one can measure round-trip time to with
> picosecond precision, using a crypto-key sequence that cannot be
> deduced from received signals fast enough to matter, thus allowing
> too-distant fobs to be ignored with sufficiently low leakage to make
> this kind of attack unprofitable.

Why would you need ps accuracy in measuring round trip time? The distance you are trying to discriminate is around 100 feet, so 200 ns which can be done with a 40 MHz oscillator. There is no need to know to the foot, how far away the key is. Just is it more than 100 feet or so.

What I'm more worried about is the damn thing opening a door inadvertently while I'm still not far away. I returned to the car from shopping and while I was putting things in the trunk, someone was admiring the gull-wing door that was open. I must have pressed the button on the key fob while it was in my pocket. That can happen from quite a distance too.

--

Rick C.

--- Get 1,000 miles of free Supercharging
--- Tesla referral code - https://ts.la/richard11209

Re: Faraday bags and other scarey things

<oghuqhpsllr02lijmmun7vk6f8fjio4oo1@4ax.com>

  copy mid

https://www.novabbs.com/tech/article-flat.php?id=113346&group=sci.electronics.design#113346

  copy link   Newsgroups: sci.electronics.design
Path: i2pn2.org!i2pn.org!weretis.net!feeder8.news.weretis.net!feeder1.feed.usenet.farm!feed.usenet.farm!tr3.eu1.usenetexpress.com!feeder.usenetexpress.com!tr3.iad1.usenetexpress.com!69.80.99.23.MISMATCH!Xl.tags.giganews.com!local-2.nntp.ord.giganews.com!nntp.supernews.com!news.supernews.com.POSTED!not-for-mail
NNTP-Posting-Date: Fri, 30 Dec 2022 20:13:19 +0000
From: jlar...@highland_atwork_technology.com (John Larkin)
Newsgroups: sci.electronics.design
Subject: Re: Faraday bags and other scarey things
Date: Fri, 30 Dec 2022 12:13:19 -0800
Organization: Highland Tech
Reply-To: xx@yy.com
Message-ID: <oghuqhpsllr02lijmmun7vk6f8fjio4oo1@4ax.com>
References: <0jbpqh9glc5nnc353uanjt33165o4reqao@4ax.com> <toieom$2r24$1@dont-email.me> <43kpqhptg3bu556300bqemp473to7kvml2@4ax.com> <0cmpqhhgqe6jv3kltna5broqnooj1rbd4i@4ax.com> <sk2uqhtep8iojr9v1r45dobfp928m0cpmr@4ax.com> <cucuqh19jgbdvclqam5m2le2227ve4gcr6@4ax.com>
X-Newsreader: Forte Agent 3.1/32.783
MIME-Version: 1.0
Content-Type: text/plain; charset=us-ascii
Content-Transfer-Encoding: 7bit
Lines: 115
X-Trace: sv3-i6GXwtgsIZf4ozLtIC2f1mOCBYfrnbZ/pyRSeP/jPE/ikjso8ftTCLhMiZQmtFFN2HpnuNqDPvhPJ3E!b9y/m1WQnHgEHTnbYifnK914TsOQsx16PozCud1spCm5XvZ4o2oZZf7a9sgTRNxWmigmbji3kScZ!Liff9w==
X-Complaints-To: www.supernews.com/docs/abuse.html
X-DMCA-Complaints-To: www.supernews.com/docs/dmca.html
X-Abuse-and-DMCA-Info: Please be sure to forward a copy of ALL headers
X-Abuse-and-DMCA-Info: Otherwise we will be unable to process your complaint properly
X-Postfilter: 1.3.40
 by: John Larkin - Fri, 30 Dec 2022 20:13 UTC

On Fri, 30 Dec 2022 14:22:08 -0500, Joe Gwinn <joegwinn@comcast.net>
wrote:

>On Fri, 30 Dec 2022 07:59:23 -0800, John Larkin
><jlarkin@highlandSNIPMEtechnology.com> wrote:
>
>>On Wed, 28 Dec 2022 19:09:10 -0500, Joe Gwinn <joegwinn@comcast.net>
>>wrote:
>>
>>>On Wed, 28 Dec 2022 15:32:07 -0800, John Larkin
>>><jlarkin@highland_atwork_technology.com> wrote:
>>>
>>>>On Wed, 28 Dec 2022 22:08:22 -0000 (UTC), Sergey Kubushyn
>>>><ksi@koi8.net> wrote:
>>>>
>>>>>In sci.electronics.repair micky <NONONOmisc07@fmguy.com> wrote:
>>>>>>
>>>>>> https://www.moneyshake.com/car-finance-guides/road-safety/how-to-protect-keyless-entry-cars
>>>>>> Unlike standard cars where access without raising the alarm requires the
>>>>>> key, keyless entry models can be ?tricked? into thinking a key is
>>>>>> present through a transmitter and receiver device.
>>>>>> If your car?s fob isn?t stored safely, then it can be very easy for a
>>>>>> pair of opportune thieves to pick up the key?s radio frequency signal,
>>>>>> without having to break into your house. Luckily, however, there are
>>>>>> many ways in which you can make it extremely difficult for them to do
>>>>>> this and protect your motor.
>>>>>>
>>>>>> ---- Except, how isthere any signal to pick up unless you are in your
>>>>>> home pressing the Unlock button on the fob at the same time the thief
>>>>>> drives by? I myself dont' bother pressing fob buttons once I've left
>>>>>> the car, until I want to drive away the next day.
>>>>>>
>>>>>> https://www.wavy.com/news/local-news/norfolk/relay-devices-being-used-to-steal-locked-cars/
>>>>>
>>>>>When I'm close to my Lexus LS460L with keyfob in my pocket it greets me with
>>>>>turning on the lights under rearview mirrors.
>>>>>
>>>>>Then, I pull the door handle and it opens the door and unfolds the rearview
>>>>>mirrors. Then, I step on the brake pedal and push the car's (not keyfob)
>>>>>START button and it starts up.
>>>>>
>>>>>Also, when I'm touching a button on the trunk with my keyfob in my pocket it
>>>>>opens the trunk.
>>>>>
>>>>>None of that happens when I have that keyfob in a copper mesh Faraday
>>>>>bag/protector no matter how close I am to my car.
>>>>>
>>>>>I don't remember pushing ANY keyfob button EVER for as long as I own that
>>>>>car. I'm locking it when leaving by pressing a button on any door handle. It
>>>>>locks all doors, folds the mirrors and activates security system when a
>>>>>button on any door handle is pressed when keyfob is nearby and not in
>>>>>Faraday bag. Won't do it if the keyfob is inside the car.
>>>>>
>>>>>And this is true for ALL relatively modern cars with that START button. They
>>>>>are ALL prone to stealing by an amplifier/receiver if keyfob is not put in a
>>>>>Faraday bag. There was a video from security camera on our local Nextdoor
>>>>>showing how a top-of-the-line Mercedes-Benz was stolen from our neighbor's
>>>>>driveway in less than a minute, from a car stopping by and a perp going out
>>>>>of that car to the moment he drove away in that Mercedes.
>>>>>
>>>>>Please, don't spread that BS about no signal unless a keyfob button is
>>>>>pressed. It is simply false.
>>>>>
>>>>>---
>>>>>******************************************************************
>>>>>* KSI@home KOI8 Net < > The impossible we do immediately. *
>>>>>* Las Vegas NV, USA < > Miracles require 24-hour notice. *
>>>>>******************************************************************
>>>>
>>>>When we get home, our keys go in a Danish Butter Cookie can. Those are
>>>>pretty good shields.
>>>>
>>>>https://www.dropbox.com/s/30c6aesh4tzjg56/Z496_Can.jpg?raw=1
>>>>
>>>>https://www.dropbox.com/s/cuxn5r7yzenb8m2/Z496_Out.jpg?raw=1
>>>
>>>I bet keeping the fobs in an open top brass bowl would also work, by
>>>being a shorted turn close to the fob's antenna loop.
>>
>>I doubt it.
>>
>>Put the fob in the bowl and walk towards the car.
>
>Good idea, and now it's warm enough.
>
>The car being tested is a 2018 Honda CR-V.
>
>The bowl did reduce range if one pushed the lock/unlock buttons using
>a 9" wooden stick (versus one's finger), but not enough to solve the
>problem I'd guess.
>
>Bowl is made of spun 0.0355" thick brass, open top diameter 7.4", and
>depth 3.3". Base, also same kind of brass, is soft soldered to the
>bowl, looks to be done by hand with a big soldering iron (not a
>torch). Quite old, has no markings whatsoever, likely US made.
>
>Also tried a 12" square sheet of 0.062" thick brass - same result.
>
>A cover ought to help, but this will take some fiddling to be able to
>push the buttons through a solid metal sheet. Although I do have some
>copper insect screening that could be pressed into service. One can
>also make a bag from this screening fabric.
>
>Longer term, I think that the entire class of man-in-the-middle
>attacks on key fobs will be solved by moving to some kind of
>ultra-wideband signal, where one can measure round-trip time to with
>picosecond precision, using a crypto-key sequence that cannot be
>deduced from received signals fast enough to matter, thus allowing
>too-distant fobs to be ignored with sufficiently low leakage to make
>this kind of attack unprofitable.
>
>Joe Gwinn

Why not make people push a button on the fob to unlock the car?

Re: Faraday bags and other scarey things

<nkmuqhttt2u9nfq363q6ae98uvf8853pae@4ax.com>

  copy mid

https://www.novabbs.com/tech/article-flat.php?id=113356&group=sci.electronics.design#113356

  copy link   Newsgroups: sci.electronics.design
Path: i2pn2.org!i2pn.org!aioe.org!feeder1.feed.usenet.farm!feed.usenet.farm!tr1.eu1.usenetexpress.com!feeder.usenetexpress.com!tr2.iad1.usenetexpress.com!69.80.99.14.MISMATCH!border-1.nntp.ord.giganews.com!nntp.giganews.com!Xl.tags.giganews.com!local-2.nntp.ord.giganews.com!news.giganews.com.POSTED!not-for-mail
NNTP-Posting-Date: Fri, 30 Dec 2022 21:40:32 +0000
From: joegw...@comcast.net (Joe Gwinn)
Newsgroups: sci.electronics.design
Subject: Re: Faraday bags and other scarey things
Date: Fri, 30 Dec 2022 16:40:32 -0500
Message-ID: <nkmuqhttt2u9nfq363q6ae98uvf8853pae@4ax.com>
References: <0jbpqh9glc5nnc353uanjt33165o4reqao@4ax.com> <toieom$2r24$1@dont-email.me> <43kpqhptg3bu556300bqemp473to7kvml2@4ax.com> <0cmpqhhgqe6jv3kltna5broqnooj1rbd4i@4ax.com> <sk2uqhtep8iojr9v1r45dobfp928m0cpmr@4ax.com> <cucuqh19jgbdvclqam5m2le2227ve4gcr6@4ax.com> <oghuqhpsllr02lijmmun7vk6f8fjio4oo1@4ax.com>
User-Agent: ForteAgent/8.00.32.1272
MIME-Version: 1.0
Content-Type: text/plain; charset=us-ascii
Content-Transfer-Encoding: 7bit
Lines: 122
X-Usenet-Provider: http://www.giganews.com
X-Trace: sv3-4KgGbI80FdKHBVKEHu6KjUcnTpP/Gww+CT7h9M1dwbRRL4AoKwv61Zkm2MVS/0RM+CeIDRMrRho7oUK!wwPIHI0JzQd5RE6bOSDar4GA6GrI72Bg4ob8wRiNuJTi8FmQGJUgx/DUbuBmtgNLP19/rZE=
X-Complaints-To: abuse@giganews.com
X-DMCA-Notifications: http://www.giganews.com/info/dmca.html
X-Abuse-and-DMCA-Info: Please be sure to forward a copy of ALL headers
X-Abuse-and-DMCA-Info: Otherwise we will be unable to process your complaint properly
X-Postfilter: 1.3.40
 by: Joe Gwinn - Fri, 30 Dec 2022 21:40 UTC

On Fri, 30 Dec 2022 12:13:19 -0800, John Larkin
<jlarkin@highland_atwork_technology.com> wrote:

>On Fri, 30 Dec 2022 14:22:08 -0500, Joe Gwinn <joegwinn@comcast.net>
>wrote:
>
>>On Fri, 30 Dec 2022 07:59:23 -0800, John Larkin
>><jlarkin@highlandSNIPMEtechnology.com> wrote:
>>
>>>On Wed, 28 Dec 2022 19:09:10 -0500, Joe Gwinn <joegwinn@comcast.net>
>>>wrote:
>>>
>>>>On Wed, 28 Dec 2022 15:32:07 -0800, John Larkin
>>>><jlarkin@highland_atwork_technology.com> wrote:
>>>>
>>>>>On Wed, 28 Dec 2022 22:08:22 -0000 (UTC), Sergey Kubushyn
>>>>><ksi@koi8.net> wrote:
>>>>>
>>>>>>In sci.electronics.repair micky <NONONOmisc07@fmguy.com> wrote:
>>>>>>>
>>>>>>> https://www.moneyshake.com/car-finance-guides/road-safety/how-to-protect-keyless-entry-cars
>>>>>>> Unlike standard cars where access without raising the alarm requires the
>>>>>>> key, keyless entry models can be ?tricked? into thinking a key is
>>>>>>> present through a transmitter and receiver device.
>>>>>>> If your car?s fob isn?t stored safely, then it can be very easy for a
>>>>>>> pair of opportune thieves to pick up the key?s radio frequency signal,
>>>>>>> without having to break into your house. Luckily, however, there are
>>>>>>> many ways in which you can make it extremely difficult for them to do
>>>>>>> this and protect your motor.
>>>>>>>
>>>>>>> ---- Except, how isthere any signal to pick up unless you are in your
>>>>>>> home pressing the Unlock button on the fob at the same time the thief
>>>>>>> drives by? I myself dont' bother pressing fob buttons once I've left
>>>>>>> the car, until I want to drive away the next day.
>>>>>>>
>>>>>>> https://www.wavy.com/news/local-news/norfolk/relay-devices-being-used-to-steal-locked-cars/
>>>>>>
>>>>>>When I'm close to my Lexus LS460L with keyfob in my pocket it greets me with
>>>>>>turning on the lights under rearview mirrors.
>>>>>>
>>>>>>Then, I pull the door handle and it opens the door and unfolds the rearview
>>>>>>mirrors. Then, I step on the brake pedal and push the car's (not keyfob)
>>>>>>START button and it starts up.
>>>>>>
>>>>>>Also, when I'm touching a button on the trunk with my keyfob in my pocket it
>>>>>>opens the trunk.
>>>>>>
>>>>>>None of that happens when I have that keyfob in a copper mesh Faraday
>>>>>>bag/protector no matter how close I am to my car.
>>>>>>
>>>>>>I don't remember pushing ANY keyfob button EVER for as long as I own that
>>>>>>car. I'm locking it when leaving by pressing a button on any door handle. It
>>>>>>locks all doors, folds the mirrors and activates security system when a
>>>>>>button on any door handle is pressed when keyfob is nearby and not in
>>>>>>Faraday bag. Won't do it if the keyfob is inside the car.
>>>>>>
>>>>>>And this is true for ALL relatively modern cars with that START button. They
>>>>>>are ALL prone to stealing by an amplifier/receiver if keyfob is not put in a
>>>>>>Faraday bag. There was a video from security camera on our local Nextdoor
>>>>>>showing how a top-of-the-line Mercedes-Benz was stolen from our neighbor's
>>>>>>driveway in less than a minute, from a car stopping by and a perp going out
>>>>>>of that car to the moment he drove away in that Mercedes.
>>>>>>
>>>>>>Please, don't spread that BS about no signal unless a keyfob button is
>>>>>>pressed. It is simply false.
>>>>>>
>>>>>>---
>>>>>>******************************************************************
>>>>>>* KSI@home KOI8 Net < > The impossible we do immediately. *
>>>>>>* Las Vegas NV, USA < > Miracles require 24-hour notice. *
>>>>>>******************************************************************
>>>>>
>>>>>When we get home, our keys go in a Danish Butter Cookie can. Those are
>>>>>pretty good shields.
>>>>>
>>>>>https://www.dropbox.com/s/30c6aesh4tzjg56/Z496_Can.jpg?raw=1
>>>>>
>>>>>https://www.dropbox.com/s/cuxn5r7yzenb8m2/Z496_Out.jpg?raw=1
>>>>
>>>>I bet keeping the fobs in an open top brass bowl would also work, by
>>>>being a shorted turn close to the fob's antenna loop.
>>>
>>>I doubt it.
>>>
>>>Put the fob in the bowl and walk towards the car.
>>
>>Good idea, and now it's warm enough.
>>
>>The car being tested is a 2018 Honda CR-V.
>>
>>The bowl did reduce range if one pushed the lock/unlock buttons using
>>a 9" wooden stick (versus one's finger), but not enough to solve the
>>problem I'd guess.
>>
>>Bowl is made of spun 0.0355" thick brass, open top diameter 7.4", and
>>depth 3.3". Base, also same kind of brass, is soft soldered to the
>>bowl, looks to be done by hand with a big soldering iron (not a
>>torch). Quite old, has no markings whatsoever, likely US made.
>>
>>Also tried a 12" square sheet of 0.062" thick brass - same result.
>>
>>A cover ought to help, but this will take some fiddling to be able to
>>push the buttons through a solid metal sheet. Although I do have some
>>copper insect screening that could be pressed into service. One can
>>also make a bag from this screening fabric.
>>
>>Longer term, I think that the entire class of man-in-the-middle
>>attacks on key fobs will be solved by moving to some kind of
>>ultra-wideband signal, where one can measure round-trip time to with
>>picosecond precision, using a crypto-key sequence that cannot be
>>deduced from received signals fast enough to matter, thus allowing
>>too-distant fobs to be ignored with sufficiently low leakage to make
>>this kind of attack unprofitable.
>>
>>Joe Gwinn
>
>Why not make people push a button on the fob to unlock the car?

Because this would defeat all hands-free uses, which most customers
are quite fond of.

Joe Gwinn

Re: Faraday bags and other scarey things

<lnmuqhtc3fa8qc2v3e8h5ik0qhhm7uh6kf@4ax.com>

  copy mid

https://www.novabbs.com/tech/article-flat.php?id=113358&group=sci.electronics.design#113358

  copy link   Newsgroups: sci.electronics.design
Path: i2pn2.org!i2pn.org!usenet.blueworldhosting.com!feed1.usenet.blueworldhosting.com!peer03.iad!feed-me.highwinds-media.com!news.highwinds-media.com!Xl.tags.giganews.com!local-1.nntp.ord.giganews.com!news.giganews.com.POSTED!not-for-mail
NNTP-Posting-Date: Fri, 30 Dec 2022 21:49:36 +0000
From: joegw...@comcast.net (Joe Gwinn)
Newsgroups: sci.electronics.design
Subject: Re: Faraday bags and other scarey things
Date: Fri, 30 Dec 2022 16:49:36 -0500
Message-ID: <lnmuqhtc3fa8qc2v3e8h5ik0qhhm7uh6kf@4ax.com>
References: <0jbpqh9glc5nnc353uanjt33165o4reqao@4ax.com> <toieom$2r24$1@dont-email.me> <43kpqhptg3bu556300bqemp473to7kvml2@4ax.com> <0cmpqhhgqe6jv3kltna5broqnooj1rbd4i@4ax.com> <sk2uqhtep8iojr9v1r45dobfp928m0cpmr@4ax.com> <cucuqh19jgbdvclqam5m2le2227ve4gcr6@4ax.com> <27a4a011-c5c5-4a64-9ed1-2ae880b94602n@googlegroups.com>
User-Agent: ForteAgent/8.00.32.1272
MIME-Version: 1.0
Content-Type: text/plain; charset=us-ascii
Content-Transfer-Encoding: 7bit
Lines: 130
X-Usenet-Provider: http://www.giganews.com
X-Trace: sv3-z4lslHnNMpsFrnlfCMKyZklVvr34oWtQowm7tc6nZI8cyRnVeC63KbVbOxZ0wjVPPlnxpJq/5SI1rE/!p6uamTsYkJmKQBNzSckqsRUlZ8TFBtht7Yiy4T5zl+4XKYFQPDs0edDzNCOWwEP7wxTkPtE=
X-Complaints-To: abuse@giganews.com
X-DMCA-Notifications: http://www.giganews.com/info/dmca.html
X-Abuse-and-DMCA-Info: Please be sure to forward a copy of ALL headers
X-Abuse-and-DMCA-Info: Otherwise we will be unable to process your complaint properly
X-Postfilter: 1.3.40
X-Received-Bytes: 8273
 by: Joe Gwinn - Fri, 30 Dec 2022 21:49 UTC

On Fri, 30 Dec 2022 11:56:22 -0800 (PST), Ricky
<gnuarm.deletethisbit@gmail.com> wrote:

>On Friday, December 30, 2022 at 2:22:21 PM UTC-5, Joe Gwinn wrote:
>> On Fri, 30 Dec 2022 07:59:23 -0800, John Larkin
>> <jla...@highlandSNIPMEtechnology.com> wrote:
>>
>> >On Wed, 28 Dec 2022 19:09:10 -0500, Joe Gwinn <joeg...@comcast.net>
>> >wrote:
>> >
>> >>On Wed, 28 Dec 2022 15:32:07 -0800, John Larkin
>> >><jlarkin@highland_atwork_technology.com> wrote:
>> >>
>> >>>On Wed, 28 Dec 2022 22:08:22 -0000 (UTC), Sergey Kubushyn
>> >>><k...@koi8.net> wrote:
>> >>>
>> >>>>In sci.electronics.repair micky <NONONO...@fmguy.com> wrote:
>> >>>>>
>> >>>>> https://www.moneyshake.com/car-finance-guides/road-safety/how-to-protect-keyless-entry-cars
>> >>>>> Unlike standard cars where access without raising the alarm requires the
>> >>>>> key, keyless entry models can be ?tricked? into thinking a key is
>> >>>>> present through a transmitter and receiver device.
>> >>>>> If your car?s fob isn?t stored safely, then it can be very easy for a
>> >>>>> pair of opportune thieves to pick up the key?s radio frequency signal,
>> >>>>> without having to break into your house. Luckily, however, there are
>> >>>>> many ways in which you can make it extremely difficult for them to do
>> >>>>> this and protect your motor.
>> >>>>>
>> >>>>> ---- Except, how isthere any signal to pick up unless you are in your
>> >>>>> home pressing the Unlock button on the fob at the same time the thief
>> >>>>> drives by? I myself dont' bother pressing fob buttons once I've left
>> >>>>> the car, until I want to drive away the next day.
>> >>>>>
>> >>>>> https://www.wavy.com/news/local-news/norfolk/relay-devices-being-used-to-steal-locked-cars/
>> >>>>
>> >>>>When I'm close to my Lexus LS460L with keyfob in my pocket it greets me with
>> >>>>turning on the lights under rearview mirrors.
>> >>>>
>> >>>>Then, I pull the door handle and it opens the door and unfolds the rearview
>> >>>>mirrors. Then, I step on the brake pedal and push the car's (not keyfob)
>> >>>>START button and it starts up.
>> >>>>
>> >>>>Also, when I'm touching a button on the trunk with my keyfob in my pocket it
>> >>>>opens the trunk.
>> >>>>
>> >>>>None of that happens when I have that keyfob in a copper mesh Faraday
>> >>>>bag/protector no matter how close I am to my car.
>> >>>>
>> >>>>I don't remember pushing ANY keyfob button EVER for as long as I own that
>> >>>>car. I'm locking it when leaving by pressing a button on any door handle. It
>> >>>>locks all doors, folds the mirrors and activates security system when a
>> >>>>button on any door handle is pressed when keyfob is nearby and not in
>> >>>>Faraday bag. Won't do it if the keyfob is inside the car.
>> >>>>
>> >>>>And this is true for ALL relatively modern cars with that START button. They
>> >>>>are ALL prone to stealing by an amplifier/receiver if keyfob is not put in a
>> >>>>Faraday bag. There was a video from security camera on our local Nextdoor
>> >>>>showing how a top-of-the-line Mercedes-Benz was stolen from our neighbor's
>> >>>>driveway in less than a minute, from a car stopping by and a perp going out
>> >>>>of that car to the moment he drove away in that Mercedes.
>> >>>>
>> >>>>Please, don't spread that BS about no signal unless a keyfob button is
>> >>>>pressed. It is simply false.
>> >>>>
>> >>>>---
>> >>>>******************************************************************
>> >>>>* KSI@home KOI8 Net < > The impossible we do immediately. *
>> >>>>* Las Vegas NV, USA < > Miracles require 24-hour notice. *
>> >>>>******************************************************************
>> >>>
>> >>>When we get home, our keys go in a Danish Butter Cookie can. Those are
>> >>>pretty good shields.
>> >>>
>> >>>https://www.dropbox.com/s/30c6aesh4tzjg56/Z496_Can.jpg?raw=1
>> >>>
>> >>>https://www.dropbox.com/s/cuxn5r7yzenb8m2/Z496_Out.jpg?raw=1
>> >>
>> >>I bet keeping the fobs in an open top brass bowl would also work, by
>> >>being a shorted turn close to the fob's antenna loop.
>> >
>> >I doubt it.
>> >
>> >Put the fob in the bowl and walk towards the car.
>> Good idea, and now it's warm enough.
>>
>> The car being tested is a 2018 Honda CR-V.
>>
>> The bowl did reduce range if one pushed the lock/unlock buttons using
>> a 9" wooden stick (versus one's finger), but not enough to solve the
>> problem I'd guess.
>>
>> Bowl is made of spun 0.0355" thick brass, open top diameter 7.4", and
>> depth 3.3". Base, also same kind of brass, is soft soldered to the
>> bowl, looks to be done by hand with a big soldering iron (not a
>> torch). Quite old, has no markings whatsoever, likely US made.
>>
>> Also tried a 12" square sheet of 0.062" thick brass - same result.
>>
>> A cover ought to help, but this will take some fiddling to be able to
>> push the buttons through a solid metal sheet. Although I do have some
>> copper insect screening that could be pressed into service. One can
>> also make a bag from this screening fabric.
>>
>> Longer term, I think that the entire class of man-in-the-middle
>> attacks on key fobs will be solved by moving to some kind of
>> ultra-wideband signal, where one can measure round-trip time to with
>> picosecond precision, using a crypto-key sequence that cannot be
>> deduced from received signals fast enough to matter, thus allowing
>> too-distant fobs to be ignored with sufficiently low leakage to make
>> this kind of attack unprofitable.
>
>Why would you need ps accuracy in measuring round trip time? The distance you are trying to discriminate is around 100 feet, so 200 ns which can be done with a 40 MHz oscillator. There is no need to know to the foot, how far away the key is. Just is it more than 100 feet or so.

The distance from my kitchen to where the cars are parked outside (and
thus accessible to would-be thieves) is well less than 100 feet.

And picoseconds are easy for UWB technology.

There is actually a use case that requires knowing the range: For
remote-start (very useful in cold places), the range may well be
large, but this does not unlock the car, so one can require no more
than six feet for opening and two or three feet for driving away.

>What I'm more worried about is the damn thing opening a door inadvertently while I'm still not far away. I returned to the car from shopping and while I was putting things in the trunk, someone was admiring the gull-wing door that was open. I must have pressed the button on the key fob while it was in my pocket. That can happen from quite a distance too.

I've had that problem, and what mostly works is a silicon-rubber
pull-on case, which makes the buttons far harder to activate.

Joe Gwinn

Re: Faraday bags and other scarey things

<6c3304c5-b054-471b-8fe2-2eac853219f5n@googlegroups.com>

  copy mid

https://www.novabbs.com/tech/article-flat.php?id=113363&group=sci.electronics.design#113363

  copy link   Newsgroups: sci.electronics.design
X-Received: by 2002:a05:620a:4895:b0:6ff:c8a2:528f with SMTP id ea21-20020a05620a489500b006ffc8a2528fmr1235679qkb.375.1672449613724;
Fri, 30 Dec 2022 17:20:13 -0800 (PST)
X-Received: by 2002:a05:620a:1526:b0:702:e7e:1d40 with SMTP id
n6-20020a05620a152600b007020e7e1d40mr1604324qkk.97.1672449613368; Fri, 30 Dec
2022 17:20:13 -0800 (PST)
Path: i2pn2.org!i2pn.org!usenet.blueworldhosting.com!feed1.usenet.blueworldhosting.com!peer02.iad!feed-me.highwinds-media.com!news.highwinds-media.com!news-out.google.com!nntp.google.com!postnews.google.com!google-groups.googlegroups.com!not-for-mail
Newsgroups: sci.electronics.design
Date: Fri, 30 Dec 2022 17:20:13 -0800 (PST)
In-Reply-To: <lnmuqhtc3fa8qc2v3e8h5ik0qhhm7uh6kf@4ax.com>
Injection-Info: google-groups.googlegroups.com; posting-host=63.114.57.174; posting-account=I-_H_woAAAA9zzro6crtEpUAyIvzd19b
NNTP-Posting-Host: 63.114.57.174
References: <0jbpqh9glc5nnc353uanjt33165o4reqao@4ax.com> <toieom$2r24$1@dont-email.me>
<43kpqhptg3bu556300bqemp473to7kvml2@4ax.com> <0cmpqhhgqe6jv3kltna5broqnooj1rbd4i@4ax.com>
<sk2uqhtep8iojr9v1r45dobfp928m0cpmr@4ax.com> <cucuqh19jgbdvclqam5m2le2227ve4gcr6@4ax.com>
<27a4a011-c5c5-4a64-9ed1-2ae880b94602n@googlegroups.com> <lnmuqhtc3fa8qc2v3e8h5ik0qhhm7uh6kf@4ax.com>
User-Agent: G2/1.0
MIME-Version: 1.0
Message-ID: <6c3304c5-b054-471b-8fe2-2eac853219f5n@googlegroups.com>
Subject: Re: Faraday bags and other scarey things
From: gnuarm.d...@gmail.com (Ricky)
Injection-Date: Sat, 31 Dec 2022 01:20:13 +0000
Content-Type: text/plain; charset="UTF-8"
Content-Transfer-Encoding: quoted-printable
X-Received-Bytes: 6054
 by: Ricky - Sat, 31 Dec 2022 01:20 UTC

On Friday, December 30, 2022 at 4:49:47 PM UTC-5, Joe Gwinn wrote:
> On Fri, 30 Dec 2022 11:56:22 -0800 (PST), Ricky
> <gnuarm.del...@gmail.com> wrote:
>
> >On Friday, December 30, 2022 at 2:22:21 PM UTC-5, Joe Gwinn wrote:
> >>
> >> Longer term, I think that the entire class of man-in-the-middle
> >> attacks on key fobs will be solved by moving to some kind of
> >> ultra-wideband signal, where one can measure round-trip time to with
> >> picosecond precision, using a crypto-key sequence that cannot be
> >> deduced from received signals fast enough to matter, thus allowing
> >> too-distant fobs to be ignored with sufficiently low leakage to make
> >> this kind of attack unprofitable.
> >
> >Why would you need ps accuracy in measuring round trip time? The distance you are trying to discriminate is around 100 feet, so 200 ns which can be done with a 40 MHz oscillator. There is no need to know to the foot, how far away the key is. Just is it more than 100 feet or so.
> The distance from my kitchen to where the cars are parked outside (and
> thus accessible to would-be thieves) is well less than 100 feet.

You are saying you need what distance? What timing. Clearly, you don't need fractions of a foot, so not ps.

> And picoseconds are easy for UWB technology.

Who cares? It's not needed.

> There is actually a use case that requires knowing the range: For
> remote-start (very useful in cold places), the range may well be
> large, but this does not unlock the car, so one can require no more
> than six feet for opening and two or three feet for driving away.

Actually, my car automatically opens the car door when I approach. From the rear, the door opens when I am about at the rear of the car. From the front, it opens when I'm about 3 feet, and only opens a crack, or it would be in my way. Then as I walk past the door, it opens the rest of the way. Very handy when my hands are full.

So I guess they've solved this issue without UWB tech. The battery in the fob lasts around a year.

> >What I'm more worried about is the damn thing opening a door inadvertently while I'm still not far away. I returned to the car from shopping and while I was putting things in the trunk, someone was admiring the gull-wing door that was open. I must have pressed the button on the key fob while it was in my pocket. That can happen from quite a distance too.
> I've had that problem, and what mostly works is a silicon-rubber
> pull-on case, which makes the buttons far harder to activate.

The buttons are already hard to press. They aren't buttons in the sense of a knobby thing sticking up. The fob is shaped like a car and you press the appropriate surface to open doors and trunks. Where the buttons are is not all that obvious and they can be hard to press because you aren't in the center of the button. It's another one of the Musk "cool" features that don't work very well.

If you removed all the pointless gimmicks from the car, the many things that don't work very well, it would be a nice car. But they stumble over their own feet with a lot of the beta features that just don't work well enough to be ready for prime time.

The other big thing, in my opinion, is the huge touch panel. I love having a big map to view, but it is so much better to have buttons and knobs for the basic things like the heating. I like the autopilot, because it gives me enough time to find the buttons on the screen and get them pressed. When you have a touch panel that has to distinguish a touch from a swipe, that is hard to do in a car, when it's bouncing around. You can use a phone, because you can brace your hand at the phone. I have to use my arm extended to reach the touch panel, resulting in the hand moving up and down with the road. Not every road is a glassy superhighway.

I think at some point, either the government will restrict the use of touch panels, or consumers will, by buying cars with a less techy interface.

Most of the controls get used once in a blue moon. The heater/AC and radio.... I mean, entertainment center, get used on every drive.

Are other cars this hard to use? My Kia has a lot of the needed controls on the steering wheel.

--

Rick C.

--+ Get 1,000 miles of free Supercharging
--+ Tesla referral code - https://ts.la/richard11209

Re: Faraday bags and other scarey things

<1ce0a10c-797b-48fa-a364-a8bf1eb581d6n@googlegroups.com>

  copy mid

https://www.novabbs.com/tech/article-flat.php?id=113370&group=sci.electronics.design#113370

  copy link   Newsgroups: sci.electronics.design
X-Received: by 2002:ac8:7219:0:b0:3a8:2e0b:7715 with SMTP id a25-20020ac87219000000b003a82e0b7715mr1153217qtp.416.1672462842014;
Fri, 30 Dec 2022 21:00:42 -0800 (PST)
X-Received: by 2002:a05:6214:2f04:b0:531:80db:b4a9 with SMTP id
od4-20020a0562142f0400b0053180dbb4a9mr1030552qvb.81.1672462841750; Fri, 30
Dec 2022 21:00:41 -0800 (PST)
Path: i2pn2.org!i2pn.org!usenet.blueworldhosting.com!feed1.usenet.blueworldhosting.com!peer01.iad!feed-me.highwinds-media.com!news.highwinds-media.com!news-out.google.com!nntp.google.com!postnews.google.com!google-groups.googlegroups.com!not-for-mail
Newsgroups: sci.electronics.design
Date: Fri, 30 Dec 2022 21:00:41 -0800 (PST)
In-Reply-To: <sk2uqhtep8iojr9v1r45dobfp928m0cpmr@4ax.com>
Injection-Info: google-groups.googlegroups.com; posting-host=209.221.140.126; posting-account=vKQm_QoAAADOaDCYsqOFDAW8NJ8sFHoE
NNTP-Posting-Host: 209.221.140.126
References: <0jbpqh9glc5nnc353uanjt33165o4reqao@4ax.com> <toieom$2r24$1@dont-email.me>
<43kpqhptg3bu556300bqemp473to7kvml2@4ax.com> <0cmpqhhgqe6jv3kltna5broqnooj1rbd4i@4ax.com>
<sk2uqhtep8iojr9v1r45dobfp928m0cpmr@4ax.com>
User-Agent: G2/1.0
MIME-Version: 1.0
Message-ID: <1ce0a10c-797b-48fa-a364-a8bf1eb581d6n@googlegroups.com>
Subject: Re: Faraday bags and other scarey things
From: whit...@gmail.com (whit3rd)
Injection-Date: Sat, 31 Dec 2022 05:00:42 +0000
Content-Type: text/plain; charset="UTF-8"
X-Received-Bytes: 1926
 by: whit3rd - Sat, 31 Dec 2022 05:00 UTC

On Friday, December 30, 2022 at 7:59:34 AM UTC-8, John Larkin wrote:
> On Wed, 28 Dec 2022 19:09:10 -0500, Joe Gwinn <joeg...@comcast.net>
> wrote:

> >I bet keeping the fobs in an open top brass bowl would also work, by
> >being a shorted turn close to the fob's antenna loop.
>
> I doubt it.
>
> Put the fob in the bowl and walk towards the car.

Oh, don't doubt it. Metal in proximity is all it takes to bollix a signal. At the metal surface, E
field is zero.
I missed a lot of phone messages once, because I left my cell phone on a copper-clad counter.
Felt a little silly when I was told no one could get in touch.

Re: Faraday bags and other scarey things

<801e0792-e22c-5dc7-a404-3a0d6eb2cb3d@electrooptical.net>

  copy mid

https://www.novabbs.com/tech/article-flat.php?id=113376&group=sci.electronics.design#113376

  copy link   Newsgroups: sci.electronics.design
Path: i2pn2.org!i2pn.org!weretis.net!feeder8.news.weretis.net!newsreader4.netcologne.de!news.netcologne.de!peer03.ams1!peer.ams1.xlned.com!news.xlned.com!peer02.iad!feed-me.highwinds-media.com!news.highwinds-media.com!Xl.tags.giganews.com!local-1.nntp.ord.giganews.com!nntp.supernews.com!news.supernews.com.POSTED!not-for-mail
NNTP-Posting-Date: Sat, 31 Dec 2022 15:51:35 +0000
Subject: Re: Faraday bags and other scarey things
Newsgroups: sci.electronics.design
References: <0jbpqh9glc5nnc353uanjt33165o4reqao@4ax.com>
<toieom$2r24$1@dont-email.me> <43kpqhptg3bu556300bqemp473to7kvml2@4ax.com>
<0cmpqhhgqe6jv3kltna5broqnooj1rbd4i@4ax.com>
<sk2uqhtep8iojr9v1r45dobfp928m0cpmr@4ax.com>
<cucuqh19jgbdvclqam5m2le2227ve4gcr6@4ax.com>
<oghuqhpsllr02lijmmun7vk6f8fjio4oo1@4ax.com>
From: pcdhSpam...@electrooptical.net (Phil Hobbs)
Message-ID: <801e0792-e22c-5dc7-a404-3a0d6eb2cb3d@electrooptical.net>
Date: Sat, 31 Dec 2022 10:51:35 -0500
User-Agent: Mozilla/5.0 (X11; Linux x86_64; rv:78.0) Gecko/20100101
Thunderbird/78.0
MIME-Version: 1.0
In-Reply-To: <oghuqhpsllr02lijmmun7vk6f8fjio4oo1@4ax.com>
Content-Type: text/plain; charset=UTF-8; format=flowed
Content-Transfer-Encoding: 7bit
Lines: 133
X-Trace: sv3-fO2wADcdhkrnWdAZ0xuqaCQBllDIUgbv12YUEt2vy0T9rISTxZ8SG/hyUlsNzb9W61RgEbsEjAEofU5!f5rKKM8AsTG9/FnGs3FWnQVtnV6JXFyqpbGrUrQT4DFXIwVtq+4eFH+MW+L/DgwRGXhDUhHWcP+w!8Ukxi9S4GtKpZeYU/CvR3g==
X-Complaints-To: www.supernews.com/docs/abuse.html
X-DMCA-Complaints-To: www.supernews.com/docs/dmca.html
X-Abuse-and-DMCA-Info: Please be sure to forward a copy of ALL headers
X-Abuse-and-DMCA-Info: Otherwise we will be unable to process your complaint properly
X-Postfilter: 1.3.40
X-Received-Bytes: 7500
 by: Phil Hobbs - Sat, 31 Dec 2022 15:51 UTC

John Larkin wrote:
> On Fri, 30 Dec 2022 14:22:08 -0500, Joe Gwinn <joegwinn@comcast.net>
> wrote:
>
>> On Fri, 30 Dec 2022 07:59:23 -0800, John Larkin
>> <jlarkin@highlandSNIPMEtechnology.com> wrote:
>>
>>> On Wed, 28 Dec 2022 19:09:10 -0500, Joe Gwinn <joegwinn@comcast.net>
>>> wrote:
>>>
>>>> On Wed, 28 Dec 2022 15:32:07 -0800, John Larkin
>>>> <jlarkin@highland_atwork_technology.com> wrote:
>>>>
>>>>> On Wed, 28 Dec 2022 22:08:22 -0000 (UTC), Sergey Kubushyn
>>>>> <ksi@koi8.net> wrote:
>>>>>
>>>>>> In sci.electronics.repair micky <NONONOmisc07@fmguy.com> wrote:
>>>>>>>
>>>>>>> https://www.moneyshake.com/car-finance-guides/road-safety/how-to-protect-keyless-entry-cars
>>>>>>> Unlike standard cars where access without raising the alarm requires the
>>>>>>> key, keyless entry models can be ?tricked? into thinking a key is
>>>>>>> present through a transmitter and receiver device.
>>>>>>> If your car?s fob isn?t stored safely, then it can be very easy for a
>>>>>>> pair of opportune thieves to pick up the key?s radio frequency signal,
>>>>>>> without having to break into your house. Luckily, however, there are
>>>>>>> many ways in which you can make it extremely difficult for them to do
>>>>>>> this and protect your motor.
>>>>>>>
>>>>>>> ---- Except, how isthere any signal to pick up unless you are in your
>>>>>>> home pressing the Unlock button on the fob at the same time the thief
>>>>>>> drives by? I myself dont' bother pressing fob buttons once I've left
>>>>>>> the car, until I want to drive away the next day.
>>>>>>>
>>>>>>> https://www.wavy.com/news/local-news/norfolk/relay-devices-being-used-to-steal-locked-cars/
>>>>>>
>>>>>> When I'm close to my Lexus LS460L with keyfob in my pocket it greets me with
>>>>>> turning on the lights under rearview mirrors.
>>>>>>
>>>>>> Then, I pull the door handle and it opens the door and unfolds the rearview
>>>>>> mirrors. Then, I step on the brake pedal and push the car's (not keyfob)
>>>>>> START button and it starts up.
>>>>>>
>>>>>> Also, when I'm touching a button on the trunk with my keyfob in my pocket it
>>>>>> opens the trunk.
>>>>>>
>>>>>> None of that happens when I have that keyfob in a copper mesh Faraday
>>>>>> bag/protector no matter how close I am to my car.
>>>>>>
>>>>>> I don't remember pushing ANY keyfob button EVER for as long as I own that
>>>>>> car. I'm locking it when leaving by pressing a button on any door handle. It
>>>>>> locks all doors, folds the mirrors and activates security system when a
>>>>>> button on any door handle is pressed when keyfob is nearby and not in
>>>>>> Faraday bag. Won't do it if the keyfob is inside the car.
>>>>>>
>>>>>> And this is true for ALL relatively modern cars with that START button. They
>>>>>> are ALL prone to stealing by an amplifier/receiver if keyfob is not put in a
>>>>>> Faraday bag. There was a video from security camera on our local Nextdoor
>>>>>> showing how a top-of-the-line Mercedes-Benz was stolen from our neighbor's
>>>>>> driveway in less than a minute, from a car stopping by and a perp going out
>>>>>> of that car to the moment he drove away in that Mercedes.
>>>>>>
>>>>>> Please, don't spread that BS about no signal unless a keyfob button is
>>>>>> pressed. It is simply false.
>>>>>>
>>>>>> ---
>>>>>> ******************************************************************
>>>>>> * KSI@home KOI8 Net < > The impossible we do immediately. *
>>>>>> * Las Vegas NV, USA < > Miracles require 24-hour notice. *
>>>>>> ******************************************************************
>>>>>
>>>>> When we get home, our keys go in a Danish Butter Cookie can. Those are
>>>>> pretty good shields.
>>>>>
>>>>> https://www.dropbox.com/s/30c6aesh4tzjg56/Z496_Can.jpg?raw=1
>>>>>
>>>>> https://www.dropbox.com/s/cuxn5r7yzenb8m2/Z496_Out.jpg?raw=1
>>>>
>>>> I bet keeping the fobs in an open top brass bowl would also work, by
>>>> being a shorted turn close to the fob's antenna loop.
>>>
>>> I doubt it.
>>>
>>> Put the fob in the bowl and walk towards the car.
>>
>> Good idea, and now it's warm enough.
>>
>> The car being tested is a 2018 Honda CR-V.
>>
>> The bowl did reduce range if one pushed the lock/unlock buttons using
>> a 9" wooden stick (versus one's finger), but not enough to solve the
>> problem I'd guess.
>>
>> Bowl is made of spun 0.0355" thick brass, open top diameter 7.4", and
>> depth 3.3". Base, also same kind of brass, is soft soldered to the
>> bowl, looks to be done by hand with a big soldering iron (not a
>> torch). Quite old, has no markings whatsoever, likely US made.
>>
>> Also tried a 12" square sheet of 0.062" thick brass - same result.
>>
>> A cover ought to help, but this will take some fiddling to be able to
>> push the buttons through a solid metal sheet. Although I do have some
>> copper insect screening that could be pressed into service. One can
>> also make a bag from this screening fabric.
>>
>> Longer term, I think that the entire class of man-in-the-middle
>> attacks on key fobs will be solved by moving to some kind of
>> ultra-wideband signal, where one can measure round-trip time to with
>> picosecond precision, using a crypto-key sequence that cannot be
>> deduced from received signals fast enough to matter, thus allowing
>> too-distant fobs to be ignored with sufficiently low leakage to make
>> this kind of attack unprofitable.
>>
>> Joe Gwinn
>
> Why not make people push a button on the fob to unlock the car?
>

Yeah, talk about solving a non-problem.

Cheers

Phil Hobbs

--
Dr Philip C D Hobbs
Principal Consultant
ElectroOptical Innovations LLC / Hobbs ElectroOptics
Optics, Electro-optics, Photonics, Analog Electronics
Briarcliff Manor NY 10510

http://electrooptical.net
http://hobbs-eo.com

Re: Faraday bags and other scarey things

<8gn0rhtcjgo1cfgqjlsthplu6svhtgiuhm@4ax.com>

  copy mid

https://www.novabbs.com/tech/article-flat.php?id=113379&group=sci.electronics.design#113379

  copy link   Newsgroups: sci.electronics.design
Path: i2pn2.org!i2pn.org!aioe.org!news.uzoreto.com!tr2.eu1.usenetexpress.com!feeder.usenetexpress.com!tr1.iad1.usenetexpress.com!69.80.99.26.MISMATCH!Xl.tags.giganews.com!local-2.nntp.ord.giganews.com!nntp.supernews.com!news.supernews.com.POSTED!not-for-mail
NNTP-Posting-Date: Sat, 31 Dec 2022 16:07:05 +0000
From: jlar...@highlandSNIPMEtechnology.com (John Larkin)
Newsgroups: sci.electronics.design
Subject: Re: Faraday bags and other scarey things
Date: Sat, 31 Dec 2022 08:07:07 -0800
Organization: Highland Tech
Reply-To: xx@yy.com
Message-ID: <8gn0rhtcjgo1cfgqjlsthplu6svhtgiuhm@4ax.com>
References: <0jbpqh9glc5nnc353uanjt33165o4reqao@4ax.com> <toieom$2r24$1@dont-email.me> <43kpqhptg3bu556300bqemp473to7kvml2@4ax.com> <0cmpqhhgqe6jv3kltna5broqnooj1rbd4i@4ax.com> <sk2uqhtep8iojr9v1r45dobfp928m0cpmr@4ax.com> <cucuqh19jgbdvclqam5m2le2227ve4gcr6@4ax.com> <oghuqhpsllr02lijmmun7vk6f8fjio4oo1@4ax.com> <nkmuqhttt2u9nfq363q6ae98uvf8853pae@4ax.com>
X-Newsreader: Forte Agent 3.1/32.783
MIME-Version: 1.0
Content-Type: text/plain; charset=us-ascii
Content-Transfer-Encoding: 7bit
Lines: 128
X-Trace: sv3-k1OK/KC52jAkrIzHBVeIeQbrtNCtuWj2+NbZRQo43oTxME6G0hHO5Pws5p2mLLl//Kv1YIr/9uaqPoJ!auUth7JtYd9JolTF9LTe/ch/zhl6dJdZaZv0PW7RH1KceU7o7lTT4pCzMgdPDWmPuYobDXAIXiXZ!v+YmBA==
X-Complaints-To: www.supernews.com/docs/abuse.html
X-DMCA-Complaints-To: www.supernews.com/docs/dmca.html
X-Abuse-and-DMCA-Info: Please be sure to forward a copy of ALL headers
X-Abuse-and-DMCA-Info: Otherwise we will be unable to process your complaint properly
X-Postfilter: 1.3.40
 by: John Larkin - Sat, 31 Dec 2022 16:07 UTC

On Fri, 30 Dec 2022 16:40:32 -0500, Joe Gwinn <joegwinn@comcast.net>
wrote:

>On Fri, 30 Dec 2022 12:13:19 -0800, John Larkin
><jlarkin@highland_atwork_technology.com> wrote:
>
>>On Fri, 30 Dec 2022 14:22:08 -0500, Joe Gwinn <joegwinn@comcast.net>
>>wrote:
>>
>>>On Fri, 30 Dec 2022 07:59:23 -0800, John Larkin
>>><jlarkin@highlandSNIPMEtechnology.com> wrote:
>>>
>>>>On Wed, 28 Dec 2022 19:09:10 -0500, Joe Gwinn <joegwinn@comcast.net>
>>>>wrote:
>>>>
>>>>>On Wed, 28 Dec 2022 15:32:07 -0800, John Larkin
>>>>><jlarkin@highland_atwork_technology.com> wrote:
>>>>>
>>>>>>On Wed, 28 Dec 2022 22:08:22 -0000 (UTC), Sergey Kubushyn
>>>>>><ksi@koi8.net> wrote:
>>>>>>
>>>>>>>In sci.electronics.repair micky <NONONOmisc07@fmguy.com> wrote:
>>>>>>>>
>>>>>>>> https://www.moneyshake.com/car-finance-guides/road-safety/how-to-protect-keyless-entry-cars
>>>>>>>> Unlike standard cars where access without raising the alarm requires the
>>>>>>>> key, keyless entry models can be ?tricked? into thinking a key is
>>>>>>>> present through a transmitter and receiver device.
>>>>>>>> If your car?s fob isn?t stored safely, then it can be very easy for a
>>>>>>>> pair of opportune thieves to pick up the key?s radio frequency signal,
>>>>>>>> without having to break into your house. Luckily, however, there are
>>>>>>>> many ways in which you can make it extremely difficult for them to do
>>>>>>>> this and protect your motor.
>>>>>>>>
>>>>>>>> ---- Except, how isthere any signal to pick up unless you are in your
>>>>>>>> home pressing the Unlock button on the fob at the same time the thief
>>>>>>>> drives by? I myself dont' bother pressing fob buttons once I've left
>>>>>>>> the car, until I want to drive away the next day.
>>>>>>>>
>>>>>>>> https://www.wavy.com/news/local-news/norfolk/relay-devices-being-used-to-steal-locked-cars/
>>>>>>>
>>>>>>>When I'm close to my Lexus LS460L with keyfob in my pocket it greets me with
>>>>>>>turning on the lights under rearview mirrors.
>>>>>>>
>>>>>>>Then, I pull the door handle and it opens the door and unfolds the rearview
>>>>>>>mirrors. Then, I step on the brake pedal and push the car's (not keyfob)
>>>>>>>START button and it starts up.
>>>>>>>
>>>>>>>Also, when I'm touching a button on the trunk with my keyfob in my pocket it
>>>>>>>opens the trunk.
>>>>>>>
>>>>>>>None of that happens when I have that keyfob in a copper mesh Faraday
>>>>>>>bag/protector no matter how close I am to my car.
>>>>>>>
>>>>>>>I don't remember pushing ANY keyfob button EVER for as long as I own that
>>>>>>>car. I'm locking it when leaving by pressing a button on any door handle. It
>>>>>>>locks all doors, folds the mirrors and activates security system when a
>>>>>>>button on any door handle is pressed when keyfob is nearby and not in
>>>>>>>Faraday bag. Won't do it if the keyfob is inside the car.
>>>>>>>
>>>>>>>And this is true for ALL relatively modern cars with that START button. They
>>>>>>>are ALL prone to stealing by an amplifier/receiver if keyfob is not put in a
>>>>>>>Faraday bag. There was a video from security camera on our local Nextdoor
>>>>>>>showing how a top-of-the-line Mercedes-Benz was stolen from our neighbor's
>>>>>>>driveway in less than a minute, from a car stopping by and a perp going out
>>>>>>>of that car to the moment he drove away in that Mercedes.
>>>>>>>
>>>>>>>Please, don't spread that BS about no signal unless a keyfob button is
>>>>>>>pressed. It is simply false.
>>>>>>>
>>>>>>>---
>>>>>>>******************************************************************
>>>>>>>* KSI@home KOI8 Net < > The impossible we do immediately. *
>>>>>>>* Las Vegas NV, USA < > Miracles require 24-hour notice. *
>>>>>>>******************************************************************
>>>>>>
>>>>>>When we get home, our keys go in a Danish Butter Cookie can. Those are
>>>>>>pretty good shields.
>>>>>>
>>>>>>https://www.dropbox.com/s/30c6aesh4tzjg56/Z496_Can.jpg?raw=1
>>>>>>
>>>>>>https://www.dropbox.com/s/cuxn5r7yzenb8m2/Z496_Out.jpg?raw=1
>>>>>
>>>>>I bet keeping the fobs in an open top brass bowl would also work, by
>>>>>being a shorted turn close to the fob's antenna loop.
>>>>
>>>>I doubt it.
>>>>
>>>>Put the fob in the bowl and walk towards the car.
>>>
>>>Good idea, and now it's warm enough.
>>>
>>>The car being tested is a 2018 Honda CR-V.
>>>
>>>The bowl did reduce range if one pushed the lock/unlock buttons using
>>>a 9" wooden stick (versus one's finger), but not enough to solve the
>>>problem I'd guess.
>>>
>>>Bowl is made of spun 0.0355" thick brass, open top diameter 7.4", and
>>>depth 3.3". Base, also same kind of brass, is soft soldered to the
>>>bowl, looks to be done by hand with a big soldering iron (not a
>>>torch). Quite old, has no markings whatsoever, likely US made.
>>>
>>>Also tried a 12" square sheet of 0.062" thick brass - same result.
>>>
>>>A cover ought to help, but this will take some fiddling to be able to
>>>push the buttons through a solid metal sheet. Although I do have some
>>>copper insect screening that could be pressed into service. One can
>>>also make a bag from this screening fabric.
>>>
>>>Longer term, I think that the entire class of man-in-the-middle
>>>attacks on key fobs will be solved by moving to some kind of
>>>ultra-wideband signal, where one can measure round-trip time to with
>>>picosecond precision, using a crypto-key sequence that cannot be
>>>deduced from received signals fast enough to matter, thus allowing
>>>too-distant fobs to be ignored with sufficiently low leakage to make
>>>this kind of attack unprofitable.
>>>
>>>Joe Gwinn
>>
>>Why not make people push a button on the fob to unlock the car?
>
>Because this would defeat all hands-free uses, which most customers
>are quite fond of.
>
>Joe Gwinn

So the choice is car-free.

Re: Faraday bags and other scarey things

<shn0rhlru6puka0f34slflo8icmuul3bgd@4ax.com>

  copy mid

https://www.novabbs.com/tech/article-flat.php?id=113381&group=sci.electronics.design#113381

  copy link   Newsgroups: sci.electronics.design
Path: i2pn2.org!i2pn.org!usenet.blueworldhosting.com!feed1.usenet.blueworldhosting.com!peer03.iad!feed-me.highwinds-media.com!news.highwinds-media.com!feeder.usenetexpress.com!tr1.iad1.usenetexpress.com!69.80.99.26.MISMATCH!Xl.tags.giganews.com!local-2.nntp.ord.giganews.com!nntp.supernews.com!news.supernews.com.POSTED!not-for-mail
NNTP-Posting-Date: Sat, 31 Dec 2022 16:17:07 +0000
From: jlar...@highlandSNIPMEtechnology.com (John Larkin)
Newsgroups: sci.electronics.design
Subject: Re: Faraday bags and other scarey things
Date: Sat, 31 Dec 2022 08:17:09 -0800
Organization: Highland Tech
Reply-To: xx@yy.com
Message-ID: <shn0rhlru6puka0f34slflo8icmuul3bgd@4ax.com>
References: <0jbpqh9glc5nnc353uanjt33165o4reqao@4ax.com> <toieom$2r24$1@dont-email.me> <43kpqhptg3bu556300bqemp473to7kvml2@4ax.com> <0cmpqhhgqe6jv3kltna5broqnooj1rbd4i@4ax.com> <sk2uqhtep8iojr9v1r45dobfp928m0cpmr@4ax.com> <cucuqh19jgbdvclqam5m2le2227ve4gcr6@4ax.com> <oghuqhpsllr02lijmmun7vk6f8fjio4oo1@4ax.com> <801e0792-e22c-5dc7-a404-3a0d6eb2cb3d@electrooptical.net>
X-Newsreader: Forte Agent 3.1/32.783
MIME-Version: 1.0
Content-Type: text/plain; charset=us-ascii
Content-Transfer-Encoding: 7bit
Lines: 140
X-Trace: sv3-CieeY00LecaJ7TLEflCmdP7Il80NooM+Ddu9HNtNn7oSLL+owIO2e15mPsKDlf50G15oxVDkM01nkL1!3fBIO3q3zTYRhe/bpFUcCl0xy9N/hl3vIy+X7traV1MkXvFBi+08CedPBfOPzymr7+AOHeN8Z+MR!wtAgSQ==
X-Complaints-To: www.supernews.com/docs/abuse.html
X-DMCA-Complaints-To: www.supernews.com/docs/dmca.html
X-Abuse-and-DMCA-Info: Please be sure to forward a copy of ALL headers
X-Abuse-and-DMCA-Info: Otherwise we will be unable to process your complaint properly
X-Postfilter: 1.3.40
X-Received-Bytes: 7978
 by: John Larkin - Sat, 31 Dec 2022 16:17 UTC

On Sat, 31 Dec 2022 10:51:35 -0500, Phil Hobbs
<pcdhSpamMeSenseless@electrooptical.net> wrote:

>John Larkin wrote:
>> On Fri, 30 Dec 2022 14:22:08 -0500, Joe Gwinn <joegwinn@comcast.net>
>> wrote:
>>
>>> On Fri, 30 Dec 2022 07:59:23 -0800, John Larkin
>>> <jlarkin@highlandSNIPMEtechnology.com> wrote:
>>>
>>>> On Wed, 28 Dec 2022 19:09:10 -0500, Joe Gwinn <joegwinn@comcast.net>
>>>> wrote:
>>>>
>>>>> On Wed, 28 Dec 2022 15:32:07 -0800, John Larkin
>>>>> <jlarkin@highland_atwork_technology.com> wrote:
>>>>>
>>>>>> On Wed, 28 Dec 2022 22:08:22 -0000 (UTC), Sergey Kubushyn
>>>>>> <ksi@koi8.net> wrote:
>>>>>>
>>>>>>> In sci.electronics.repair micky <NONONOmisc07@fmguy.com> wrote:
>>>>>>>>
>>>>>>>> https://www.moneyshake.com/car-finance-guides/road-safety/how-to-protect-keyless-entry-cars
>>>>>>>> Unlike standard cars where access without raising the alarm requires the
>>>>>>>> key, keyless entry models can be ?tricked? into thinking a key is
>>>>>>>> present through a transmitter and receiver device.
>>>>>>>> If your car?s fob isn?t stored safely, then it can be very easy for a
>>>>>>>> pair of opportune thieves to pick up the key?s radio frequency signal,
>>>>>>>> without having to break into your house. Luckily, however, there are
>>>>>>>> many ways in which you can make it extremely difficult for them to do
>>>>>>>> this and protect your motor.
>>>>>>>>
>>>>>>>> ---- Except, how isthere any signal to pick up unless you are in your
>>>>>>>> home pressing the Unlock button on the fob at the same time the thief
>>>>>>>> drives by? I myself dont' bother pressing fob buttons once I've left
>>>>>>>> the car, until I want to drive away the next day.
>>>>>>>>
>>>>>>>> https://www.wavy.com/news/local-news/norfolk/relay-devices-being-used-to-steal-locked-cars/
>>>>>>>
>>>>>>> When I'm close to my Lexus LS460L with keyfob in my pocket it greets me with
>>>>>>> turning on the lights under rearview mirrors.
>>>>>>>
>>>>>>> Then, I pull the door handle and it opens the door and unfolds the rearview
>>>>>>> mirrors. Then, I step on the brake pedal and push the car's (not keyfob)
>>>>>>> START button and it starts up.
>>>>>>>
>>>>>>> Also, when I'm touching a button on the trunk with my keyfob in my pocket it
>>>>>>> opens the trunk.
>>>>>>>
>>>>>>> None of that happens when I have that keyfob in a copper mesh Faraday
>>>>>>> bag/protector no matter how close I am to my car.
>>>>>>>
>>>>>>> I don't remember pushing ANY keyfob button EVER for as long as I own that
>>>>>>> car. I'm locking it when leaving by pressing a button on any door handle. It
>>>>>>> locks all doors, folds the mirrors and activates security system when a
>>>>>>> button on any door handle is pressed when keyfob is nearby and not in
>>>>>>> Faraday bag. Won't do it if the keyfob is inside the car.
>>>>>>>
>>>>>>> And this is true for ALL relatively modern cars with that START button. They
>>>>>>> are ALL prone to stealing by an amplifier/receiver if keyfob is not put in a
>>>>>>> Faraday bag. There was a video from security camera on our local Nextdoor
>>>>>>> showing how a top-of-the-line Mercedes-Benz was stolen from our neighbor's
>>>>>>> driveway in less than a minute, from a car stopping by and a perp going out
>>>>>>> of that car to the moment he drove away in that Mercedes.
>>>>>>>
>>>>>>> Please, don't spread that BS about no signal unless a keyfob button is
>>>>>>> pressed. It is simply false.
>>>>>>>
>>>>>>> ---
>>>>>>> ******************************************************************
>>>>>>> * KSI@home KOI8 Net < > The impossible we do immediately. *
>>>>>>> * Las Vegas NV, USA < > Miracles require 24-hour notice. *
>>>>>>> ******************************************************************
>>>>>>
>>>>>> When we get home, our keys go in a Danish Butter Cookie can. Those are
>>>>>> pretty good shields.
>>>>>>
>>>>>> https://www.dropbox.com/s/30c6aesh4tzjg56/Z496_Can.jpg?raw=1
>>>>>>
>>>>>> https://www.dropbox.com/s/cuxn5r7yzenb8m2/Z496_Out.jpg?raw=1
>>>>>
>>>>> I bet keeping the fobs in an open top brass bowl would also work, by
>>>>> being a shorted turn close to the fob's antenna loop.
>>>>
>>>> I doubt it.
>>>>
>>>> Put the fob in the bowl and walk towards the car.
>>>
>>> Good idea, and now it's warm enough.
>>>
>>> The car being tested is a 2018 Honda CR-V.
>>>
>>> The bowl did reduce range if one pushed the lock/unlock buttons using
>>> a 9" wooden stick (versus one's finger), but not enough to solve the
>>> problem I'd guess.
>>>
>>> Bowl is made of spun 0.0355" thick brass, open top diameter 7.4", and
>>> depth 3.3". Base, also same kind of brass, is soft soldered to the
>>> bowl, looks to be done by hand with a big soldering iron (not a
>>> torch). Quite old, has no markings whatsoever, likely US made.
>>>
>>> Also tried a 12" square sheet of 0.062" thick brass - same result.
>>>
>>> A cover ought to help, but this will take some fiddling to be able to
>>> push the buttons through a solid metal sheet. Although I do have some
>>> copper insect screening that could be pressed into service. One can
>>> also make a bag from this screening fabric.
>>>
>>> Longer term, I think that the entire class of man-in-the-middle
>>> attacks on key fobs will be solved by moving to some kind of
>>> ultra-wideband signal, where one can measure round-trip time to with
>>> picosecond precision, using a crypto-key sequence that cannot be
>>> deduced from received signals fast enough to matter, thus allowing
>>> too-distant fobs to be ignored with sufficiently low leakage to make
>>> this kind of attack unprofitable.
>>>
>>> Joe Gwinn
>>
>> Why not make people push a button on the fob to unlock the car?
>>
>
>Yeah, talk about solving a non-problem.
>
>Cheers
>
>Phil Hobbs

My RF key fob has a little (mechanical) button that, when pressed, a
metal key pops out! That will open doors even if all involved
batteries are dead.

I got a little lithium battery jump starter as a backup. USB
rechargeable. It has USB power out too. I don't have to mess with
jumper cables to start my car, or someone else's car.

It claims 1000 amps, which is improbable, but it started my V6 Audi
when it was cold and had a bad battery.

I'm impressed at what great mechanisms are mechanical keys and locks.
And how much bad code is around.

Re: Faraday bags and other scarey things

<c6o0rht0r82pcn0htvaempt1pda0pqgsdl@4ax.com>

  copy mid

https://www.novabbs.com/tech/article-flat.php?id=113382&group=sci.electronics.design#113382

  copy link   Newsgroups: sci.electronics.design
Path: i2pn2.org!i2pn.org!usenet.blueworldhosting.com!feed1.usenet.blueworldhosting.com!peer03.iad!feed-me.highwinds-media.com!news.highwinds-media.com!feeder.usenetexpress.com!tr3.iad1.usenetexpress.com!69.80.99.27.MISMATCH!Xl.tags.giganews.com!local-2.nntp.ord.giganews.com!nntp.supernews.com!news.supernews.com.POSTED!not-for-mail
NNTP-Posting-Date: Sat, 31 Dec 2022 16:18:49 +0000
From: jlar...@highlandSNIPMEtechnology.com (John Larkin)
Newsgroups: sci.electronics.design
Subject: Re: Faraday bags and other scarey things
Date: Sat, 31 Dec 2022 08:18:50 -0800
Organization: Highland Tech
Reply-To: xx@yy.com
Message-ID: <c6o0rht0r82pcn0htvaempt1pda0pqgsdl@4ax.com>
References: <0jbpqh9glc5nnc353uanjt33165o4reqao@4ax.com> <toieom$2r24$1@dont-email.me> <43kpqhptg3bu556300bqemp473to7kvml2@4ax.com> <0cmpqhhgqe6jv3kltna5broqnooj1rbd4i@4ax.com> <sk2uqhtep8iojr9v1r45dobfp928m0cpmr@4ax.com> <1ce0a10c-797b-48fa-a364-a8bf1eb581d6n@googlegroups.com>
X-Newsreader: Forte Agent 3.1/32.783
MIME-Version: 1.0
Content-Type: text/plain; charset=us-ascii
Content-Transfer-Encoding: 7bit
Lines: 23
X-Trace: sv3-Qpu7y7bNzP/KZrQHFNJ6uDyepeiujsj+CYiI6o+b2+PttOvukiBNqubAGNKpzzOgqMSLmyIfu0mYBKF!rhEJs9Jl444j39MqWeTrmbyrEO0zKuirrS3zQ1EqzmotNYxSnjV5XBD2/R9hu+OS6RJd+ud8MtiZ!IXYk/Q==
X-Complaints-To: www.supernews.com/docs/abuse.html
X-DMCA-Complaints-To: www.supernews.com/docs/dmca.html
X-Abuse-and-DMCA-Info: Please be sure to forward a copy of ALL headers
X-Abuse-and-DMCA-Info: Otherwise we will be unable to process your complaint properly
X-Postfilter: 1.3.40
X-Received-Bytes: 2209
 by: John Larkin - Sat, 31 Dec 2022 16:18 UTC

On Fri, 30 Dec 2022 21:00:41 -0800 (PST), whit3rd <whit3rd@gmail.com>
wrote:

>On Friday, December 30, 2022 at 7:59:34 AM UTC-8, John Larkin wrote:
>> On Wed, 28 Dec 2022 19:09:10 -0500, Joe Gwinn <joeg...@comcast.net>
>> wrote:
>
>> >I bet keeping the fobs in an open top brass bowl would also work, by
>> >being a shorted turn close to the fob's antenna loop.
>>
>> I doubt it.
>>
>> Put the fob in the bowl and walk towards the car.
>
>Oh, don't doubt it. Metal in proximity is all it takes to bollix a signal. At the metal surface, E
>field is zero.

Joe tried it.

>I missed a lot of phone messages once, because I left my cell phone on a copper-clad counter.
>Felt a little silly when I was told no one could get in touch.

Re: Faraday bags and other scarey things

<1ab0342e-d340-a735-5e58-19d3648b8577@electrooptical.net>

  copy mid

https://www.novabbs.com/tech/article-flat.php?id=113387&group=sci.electronics.design#113387

  copy link   Newsgroups: sci.electronics.design
Path: i2pn2.org!i2pn.org!usenet.blueworldhosting.com!feed1.usenet.blueworldhosting.com!peer02.iad!feed-me.highwinds-media.com!news.highwinds-media.com!feeder.usenetexpress.com!tr1.iad1.usenetexpress.com!69.80.99.27.MISMATCH!Xl.tags.giganews.com!local-2.nntp.ord.giganews.com!nntp.supernews.com!news.supernews.com.POSTED!not-for-mail
NNTP-Posting-Date: Sat, 31 Dec 2022 16:47:05 +0000
Subject: Re: Faraday bags and other scarey things
Newsgroups: sci.electronics.design
References: <0jbpqh9glc5nnc353uanjt33165o4reqao@4ax.com> <toieom$2r24$1@dont-email.me> <43kpqhptg3bu556300bqemp473to7kvml2@4ax.com> <0cmpqhhgqe6jv3kltna5broqnooj1rbd4i@4ax.com> <sk2uqhtep8iojr9v1r45dobfp928m0cpmr@4ax.com> <1ce0a10c-797b-48fa-a364-a8bf1eb581d6n@googlegroups.com> <c6o0rht0r82pcn0htvaempt1pda0pqgsdl@4ax.com>
From: pcdhSpam...@electrooptical.net (Phil Hobbs)
Message-ID: <1ab0342e-d340-a735-5e58-19d3648b8577@electrooptical.net>
Date: Sat, 31 Dec 2022 11:47:04 -0500
User-Agent: Mozilla/5.0 (X11; Linux x86_64; rv:78.0) Gecko/20100101 Thunderbird/78.0
MIME-Version: 1.0
In-Reply-To: <c6o0rht0r82pcn0htvaempt1pda0pqgsdl@4ax.com>
Content-Type: text/plain; charset=UTF-8; format=flowed
Content-Transfer-Encoding: 7bit
Lines: 57
X-Trace: sv3-5l31FCsW8KiKRDa394/neRjqDW1F+8goJ3RQhSzOJAD+NZY3Jd5IHLjcFlRUc5muURcD7GFQeZ9OmUr!7GftOSkhbSVhm43tBhYmTJyQcIxhwxj5+NFtVXMP4LKCLBE0qfCmq45hMeTQ8aq4mG2om1YiaMr9!xBsif+5sPIJv+xWy2884lg==
X-Complaints-To: www.supernews.com/docs/abuse.html
X-DMCA-Complaints-To: www.supernews.com/docs/dmca.html
X-Abuse-and-DMCA-Info: Please be sure to forward a copy of ALL headers
X-Abuse-and-DMCA-Info: Otherwise we will be unable to process your complaint properly
X-Postfilter: 1.3.40
X-Received-Bytes: 3274
 by: Phil Hobbs - Sat, 31 Dec 2022 16:47 UTC

John Larkin wrote:
> On Fri, 30 Dec 2022 21:00:41 -0800 (PST), whit3rd
> <whit3rd@gmail.com> wrote:
>
>> On Friday, December 30, 2022 at 7:59:34 AM UTC-8, John Larkin
>> wrote:
>>> On Wed, 28 Dec 2022 19:09:10 -0500, Joe Gwinn
>>> <joeg...@comcast.net> wrote:
>>
>>>> I bet keeping the fobs in an open top brass bowl would also
>>>> work, by being a shorted turn close to the fob's antenna loop.
>>>
>>> I doubt it.
>>>
>>> Put the fob in the bowl and walk towards the car.
>>
>> Oh, don't doubt it. Metal in proximity is all it takes to bollix
>> a signal. At the metal surface, E field is zero.
>
> Joe tried it.
>
>
>
>> I missed a lot of phone messages once, because I left my cell phone
>> on a copper-clad counter. Felt a little silly when I was told no
>> one could get in touch.

Third has lived a sheltered life. ;)

We built a couple of test stands for our fire and spark sensors used on
cotton harvesters. The testers are built in a large die-cast aluminum
stomp box with six screws in the lid.

The Raspberry Pi inside has no trouble communicating to a nearby laptop
via wifi.

For really effective shielding, you want an uncoated paint can, an old
nickel-plated 70-mm film can, or a Danish Butter Cookies tin of the
right sort, i.e. where the printing doesn't cover the sealing edge of
the tin and lid.

Oh, or a stomp box with a metal mesh gasket.

Cheers

Phil Hobbs

--
Dr Philip C D Hobbs
Principal Consultant
ElectroOptical Innovations LLC / Hobbs ElectroOptics
Optics, Electro-optics, Photonics, Analog Electronics
Briarcliff Manor NY 10510

http://electrooptical.net
http://hobbs-eo.com

Re: Faraday bags and other scarey things

<b7q0rhpfie5i7vkce4fhq880l4030k8ug7@4ax.com>

  copy mid

https://www.novabbs.com/tech/article-flat.php?id=113390&group=sci.electronics.design#113390

  copy link   Newsgroups: sci.electronics.design
Path: i2pn2.org!i2pn.org!usenet.blueworldhosting.com!feed1.usenet.blueworldhosting.com!peer01.iad!feed-me.highwinds-media.com!news.highwinds-media.com!Xl.tags.giganews.com!local-1.nntp.ord.giganews.com!news.giganews.com.POSTED!not-for-mail
NNTP-Posting-Date: Sat, 31 Dec 2022 16:56:08 +0000
From: joegw...@comcast.net (Joe Gwinn)
Newsgroups: sci.electronics.design
Subject: Re: Faraday bags and other scarey things
Date: Sat, 31 Dec 2022 11:56:08 -0500
Message-ID: <b7q0rhpfie5i7vkce4fhq880l4030k8ug7@4ax.com>
References: <0jbpqh9glc5nnc353uanjt33165o4reqao@4ax.com> <toieom$2r24$1@dont-email.me> <43kpqhptg3bu556300bqemp473to7kvml2@4ax.com> <0cmpqhhgqe6jv3kltna5broqnooj1rbd4i@4ax.com> <sk2uqhtep8iojr9v1r45dobfp928m0cpmr@4ax.com> <cucuqh19jgbdvclqam5m2le2227ve4gcr6@4ax.com> <oghuqhpsllr02lijmmun7vk6f8fjio4oo1@4ax.com> <801e0792-e22c-5dc7-a404-3a0d6eb2cb3d@electrooptical.net> <shn0rhlru6puka0f34slflo8icmuul3bgd@4ax.com>
User-Agent: ForteAgent/8.00.32.1272
MIME-Version: 1.0
Content-Type: text/plain; charset=us-ascii
Content-Transfer-Encoding: 7bit
Lines: 156
X-Usenet-Provider: http://www.giganews.com
X-Trace: sv3-4d7q59HwDw5/kyT7CnHvQNh3VeVobC1x4UDzs1AwaqKl07oNAzh7wYXFySAKtl4bvhoDnDXnaY8GYdg!yI7ttSqVQ6l0V+s5hXoEKLkwohSg1wruS2t1hIkeV8naJETeHjPfIP6vuWsROuBH2YlAfw4=
X-Complaints-To: abuse@giganews.com
X-DMCA-Notifications: http://www.giganews.com/info/dmca.html
X-Abuse-and-DMCA-Info: Please be sure to forward a copy of ALL headers
X-Abuse-and-DMCA-Info: Otherwise we will be unable to process your complaint properly
X-Postfilter: 1.3.40
X-Received-Bytes: 8362
 by: Joe Gwinn - Sat, 31 Dec 2022 16:56 UTC

On Sat, 31 Dec 2022 08:17:09 -0800, John Larkin
<jlarkin@highlandSNIPMEtechnology.com> wrote:

>On Sat, 31 Dec 2022 10:51:35 -0500, Phil Hobbs
><pcdhSpamMeSenseless@electrooptical.net> wrote:
>
>>John Larkin wrote:
>>> On Fri, 30 Dec 2022 14:22:08 -0500, Joe Gwinn <joegwinn@comcast.net>
>>> wrote:
>>>
>>>> On Fri, 30 Dec 2022 07:59:23 -0800, John Larkin
>>>> <jlarkin@highlandSNIPMEtechnology.com> wrote:
>>>>
>>>>> On Wed, 28 Dec 2022 19:09:10 -0500, Joe Gwinn <joegwinn@comcast.net>
>>>>> wrote:
>>>>>
>>>>>> On Wed, 28 Dec 2022 15:32:07 -0800, John Larkin
>>>>>> <jlarkin@highland_atwork_technology.com> wrote:
>>>>>>
>>>>>>> On Wed, 28 Dec 2022 22:08:22 -0000 (UTC), Sergey Kubushyn
>>>>>>> <ksi@koi8.net> wrote:
>>>>>>>
>>>>>>>> In sci.electronics.repair micky <NONONOmisc07@fmguy.com> wrote:
>>>>>>>>>
>>>>>>>>> https://www.moneyshake.com/car-finance-guides/road-safety/how-to-protect-keyless-entry-cars
>>>>>>>>> Unlike standard cars where access without raising the alarm requires the
>>>>>>>>> key, keyless entry models can be ?tricked? into thinking a key is
>>>>>>>>> present through a transmitter and receiver device.
>>>>>>>>> If your car?s fob isn?t stored safely, then it can be very easy for a
>>>>>>>>> pair of opportune thieves to pick up the key?s radio frequency signal,
>>>>>>>>> without having to break into your house. Luckily, however, there are
>>>>>>>>> many ways in which you can make it extremely difficult for them to do
>>>>>>>>> this and protect your motor.
>>>>>>>>>
>>>>>>>>> ---- Except, how isthere any signal to pick up unless you are in your
>>>>>>>>> home pressing the Unlock button on the fob at the same time the thief
>>>>>>>>> drives by? I myself dont' bother pressing fob buttons once I've left
>>>>>>>>> the car, until I want to drive away the next day.
>>>>>>>>>
>>>>>>>>> https://www.wavy.com/news/local-news/norfolk/relay-devices-being-used-to-steal-locked-cars/
>>>>>>>>
>>>>>>>> When I'm close to my Lexus LS460L with keyfob in my pocket it greets me with
>>>>>>>> turning on the lights under rearview mirrors.
>>>>>>>>
>>>>>>>> Then, I pull the door handle and it opens the door and unfolds the rearview
>>>>>>>> mirrors. Then, I step on the brake pedal and push the car's (not keyfob)
>>>>>>>> START button and it starts up.
>>>>>>>>
>>>>>>>> Also, when I'm touching a button on the trunk with my keyfob in my pocket it
>>>>>>>> opens the trunk.
>>>>>>>>
>>>>>>>> None of that happens when I have that keyfob in a copper mesh Faraday
>>>>>>>> bag/protector no matter how close I am to my car.
>>>>>>>>
>>>>>>>> I don't remember pushing ANY keyfob button EVER for as long as I own that
>>>>>>>> car. I'm locking it when leaving by pressing a button on any door handle. It
>>>>>>>> locks all doors, folds the mirrors and activates security system when a
>>>>>>>> button on any door handle is pressed when keyfob is nearby and not in
>>>>>>>> Faraday bag. Won't do it if the keyfob is inside the car.
>>>>>>>>
>>>>>>>> And this is true for ALL relatively modern cars with that START button. They
>>>>>>>> are ALL prone to stealing by an amplifier/receiver if keyfob is not put in a
>>>>>>>> Faraday bag. There was a video from security camera on our local Nextdoor
>>>>>>>> showing how a top-of-the-line Mercedes-Benz was stolen from our neighbor's
>>>>>>>> driveway in less than a minute, from a car stopping by and a perp going out
>>>>>>>> of that car to the moment he drove away in that Mercedes.
>>>>>>>>
>>>>>>>> Please, don't spread that BS about no signal unless a keyfob button is
>>>>>>>> pressed. It is simply false.
>>>>>>>>
>>>>>>>> ---
>>>>>>>> ******************************************************************
>>>>>>>> * KSI@home KOI8 Net < > The impossible we do immediately. *
>>>>>>>> * Las Vegas NV, USA < > Miracles require 24-hour notice. *
>>>>>>>> ******************************************************************
>>>>>>>
>>>>>>> When we get home, our keys go in a Danish Butter Cookie can. Those are
>>>>>>> pretty good shields.
>>>>>>>
>>>>>>> https://www.dropbox.com/s/30c6aesh4tzjg56/Z496_Can.jpg?raw=1
>>>>>>>
>>>>>>> https://www.dropbox.com/s/cuxn5r7yzenb8m2/Z496_Out.jpg?raw=1
>>>>>>
>>>>>> I bet keeping the fobs in an open top brass bowl would also work, by
>>>>>> being a shorted turn close to the fob's antenna loop.
>>>>>
>>>>> I doubt it.
>>>>>
>>>>> Put the fob in the bowl and walk towards the car.
>>>>
>>>> Good idea, and now it's warm enough.
>>>>
>>>> The car being tested is a 2018 Honda CR-V.
>>>>
>>>> The bowl did reduce range if one pushed the lock/unlock buttons using
>>>> a 9" wooden stick (versus one's finger), but not enough to solve the
>>>> problem I'd guess.
>>>>
>>>> Bowl is made of spun 0.0355" thick brass, open top diameter 7.4", and
>>>> depth 3.3". Base, also same kind of brass, is soft soldered to the
>>>> bowl, looks to be done by hand with a big soldering iron (not a
>>>> torch). Quite old, has no markings whatsoever, likely US made.
>>>>
>>>> Also tried a 12" square sheet of 0.062" thick brass - same result.
>>>>
>>>> A cover ought to help, but this will take some fiddling to be able to
>>>> push the buttons through a solid metal sheet. Although I do have some
>>>> copper insect screening that could be pressed into service. One can
>>>> also make a bag from this screening fabric.
>>>>
>>>> Longer term, I think that the entire class of man-in-the-middle
>>>> attacks on key fobs will be solved by moving to some kind of
>>>> ultra-wideband signal, where one can measure round-trip time to with
>>>> picosecond precision, using a crypto-key sequence that cannot be
>>>> deduced from received signals fast enough to matter, thus allowing
>>>> too-distant fobs to be ignored with sufficiently low leakage to make
>>>> this kind of attack unprofitable.
>>>>
>>>> Joe Gwinn
>>>
>>> Why not make people push a button on the fob to unlock the car?
>>>
>>
>>Yeah, talk about solving a non-problem.
>>
>>Cheers
>>
>>Phil Hobbs
>
>My RF key fob has a little (mechanical) button that, when pressed, a
>metal key pops out! That will open doors even if all involved
>batteries are dead.

I have one of those too.

>I got a little lithium battery jump starter as a backup. USB
>rechargeable. It has USB power out too. I don't have to mess with
>jumper cables to start my car, or someone else's car.
>
>It claims 1000 amps, which is improbable, but it started my V6 Audi
>when it was cold and had a bad battery.

This sounds worthwhile. Modern cars are actually too big for their
batteries, by a factor.

What make and model?

>I'm impressed at what great mechanisms are mechanical keys and locks.
>And how much bad code is around.

Yes. Bob Pease's comment was that his best programming language was
solder.

Joe Gwinn

Re: Faraday bags and other scarey things

<d2a69c45-8384-5f69-cd90-1df94b6b1e2a@electrooptical.net>

  copy mid

https://www.novabbs.com/tech/article-flat.php?id=113392&group=sci.electronics.design#113392

  copy link   Newsgroups: sci.electronics.design
Path: i2pn2.org!i2pn.org!eternal-september.org!reader01.eternal-september.org!.POSTED!not-for-mail
From: pcdhSpam...@electrooptical.net (Phil Hobbs)
Newsgroups: sci.electronics.design
Subject: Re: Faraday bags and other scarey things
Date: Sat, 31 Dec 2022 12:21:28 -0500
Organization: A noiseless patient Spider
Lines: 59
Message-ID: <d2a69c45-8384-5f69-cd90-1df94b6b1e2a@electrooptical.net>
References: <0jbpqh9glc5nnc353uanjt33165o4reqao@4ax.com>
<toieom$2r24$1@dont-email.me> <43kpqhptg3bu556300bqemp473to7kvml2@4ax.com>
<0cmpqhhgqe6jv3kltna5broqnooj1rbd4i@4ax.com>
<sk2uqhtep8iojr9v1r45dobfp928m0cpmr@4ax.com>
<cucuqh19jgbdvclqam5m2le2227ve4gcr6@4ax.com>
<oghuqhpsllr02lijmmun7vk6f8fjio4oo1@4ax.com>
<801e0792-e22c-5dc7-a404-3a0d6eb2cb3d@electrooptical.net>
<shn0rhlru6puka0f34slflo8icmuul3bgd@4ax.com>
<b7q0rhpfie5i7vkce4fhq880l4030k8ug7@4ax.com>
MIME-Version: 1.0
Content-Type: text/plain; charset=UTF-8; format=flowed
Content-Transfer-Encoding: 7bit
Injection-Info: reader01.eternal-september.org; posting-host="0dd0c90a77c8b5442685724f4670f18b";
logging-data="1136657"; mail-complaints-to="abuse@eternal-september.org"; posting-account="U2FsdGVkX19LEnW4hB3pae8IwzAYjb6O"
User-Agent: Mozilla/5.0 (X11; Linux x86_64; rv:78.0) Gecko/20100101
Thunderbird/78.0
Cancel-Lock: sha1:KMd4v7Cz51FC3TjgRBgGVxhZzkI=
In-Reply-To: <b7q0rhpfie5i7vkce4fhq880l4030k8ug7@4ax.com>
 by: Phil Hobbs - Sat, 31 Dec 2022 17:21 UTC

Joe Gwinn wrote:
> On Sat, 31 Dec 2022 08:17:09 -0800, John Larkin
> <jlarkin@highlandSNIPMEtechnology.com> wrote:
>
>> On Sat, 31 Dec 2022 10:51:35 -0500, Phil Hobbs
>> <pcdhSpamMeSenseless@electrooptical.net> wrote:
>><snip>

>>>>
>>>> Why not make people push a button on the fob to unlock the car?
>>>>
>>>
>>> Yeah, talk about solving a non-problem.
>>>
>>> Cheers
>>>
>>> Phil Hobbs
>>
>> My RF key fob has a little (mechanical) button that, when pressed, a
>> metal key pops out! That will open doors even if all involved
>> batteries are dead.
>
> I have one of those too.
>
>
>> I got a little lithium battery jump starter as a backup. USB
>> rechargeable. It has USB power out too. I don't have to mess with
>> jumper cables to start my car, or someone else's car.
>>
>> It claims 1000 amps, which is improbable, but it started my V6 Audi
>> when it was cold and had a bad battery.
>
> This sounds worthwhile. Modern cars are actually too big for their
> batteries, by a factor.
>
> What make and model?
>

The Project Farm guy (Todd) has done a couple of jump-box comparos that
are worth watching (at 1.75x, usually).

https://www.youtube.com/watch?v=VRFwEUr_Vjc
https://www.youtube.com/watch?v=5zAgL2L4ILY

Cheers

Phil Hobbs

--
Dr Philip C D Hobbs
Principal Consultant
ElectroOptical Innovations LLC / Hobbs ElectroOptics
Optics, Electro-optics, Photonics, Analog Electronics
Briarcliff Manor NY 10510

http://electrooptical.net
http://hobbs-eo.com

Re: Faraday bags and other scarey things

<87282379-4f47-4abd-9849-ccc241aa3b6cn@googlegroups.com>

  copy mid

https://www.novabbs.com/tech/article-flat.php?id=113394&group=sci.electronics.design#113394

  copy link   Newsgroups: sci.electronics.design
X-Received: by 2002:a05:620a:8204:b0:6fa:6424:5d87 with SMTP id ow4-20020a05620a820400b006fa64245d87mr1832213qkn.651.1672508632791;
Sat, 31 Dec 2022 09:43:52 -0800 (PST)
X-Received: by 2002:a05:6214:2b9a:b0:4c7:20e7:a580 with SMTP id
kr26-20020a0562142b9a00b004c720e7a580mr1929748qvb.43.1672508632616; Sat, 31
Dec 2022 09:43:52 -0800 (PST)
Path: i2pn2.org!i2pn.org!usenet.blueworldhosting.com!feed1.usenet.blueworldhosting.com!peer03.iad!feed-me.highwinds-media.com!news.highwinds-media.com!news-out.google.com!nntp.google.com!postnews.google.com!google-groups.googlegroups.com!not-for-mail
Newsgroups: sci.electronics.design
Date: Sat, 31 Dec 2022 09:43:52 -0800 (PST)
In-Reply-To: <1ab0342e-d340-a735-5e58-19d3648b8577@electrooptical.net>
Injection-Info: google-groups.googlegroups.com; posting-host=209.221.140.126; posting-account=vKQm_QoAAADOaDCYsqOFDAW8NJ8sFHoE
NNTP-Posting-Host: 209.221.140.126
References: <0jbpqh9glc5nnc353uanjt33165o4reqao@4ax.com> <toieom$2r24$1@dont-email.me>
<43kpqhptg3bu556300bqemp473to7kvml2@4ax.com> <0cmpqhhgqe6jv3kltna5broqnooj1rbd4i@4ax.com>
<sk2uqhtep8iojr9v1r45dobfp928m0cpmr@4ax.com> <1ce0a10c-797b-48fa-a364-a8bf1eb581d6n@googlegroups.com>
<c6o0rht0r82pcn0htvaempt1pda0pqgsdl@4ax.com> <1ab0342e-d340-a735-5e58-19d3648b8577@electrooptical.net>
User-Agent: G2/1.0
MIME-Version: 1.0
Message-ID: <87282379-4f47-4abd-9849-ccc241aa3b6cn@googlegroups.com>
Subject: Re: Faraday bags and other scarey things
From: whit...@gmail.com (whit3rd)
Injection-Date: Sat, 31 Dec 2022 17:43:52 +0000
Content-Type: text/plain; charset="UTF-8"
X-Received-Bytes: 3218
 by: whit3rd - Sat, 31 Dec 2022 17:43 UTC

On Saturday, December 31, 2022 at 8:47:16 AM UTC-8, Phil Hobbs wrote:
> John Larkin wrote:
> > On Fri, 30 Dec 2022 21:00:41 -0800 (PST), whit3rd
> > <whi...@gmail.com> wrote:
> >
> >> On Friday, December 30, 2022 at 7:59:34 AM UTC-8, John Larkin
> >> wrote:
> >>> On Wed, 28 Dec 2022 19:09:10 -0500, Joe Gwinn
> >>> <joeg...@comcast.net> wrote:

> >>>> I bet keeping the fobs in an open top brass bowl would also
> >>>> work, by being a shorted turn close to the fob's antenna loop.
> >>>
> >>> I doubt it.
> >>>
> >>> Put the fob in the bowl and walk towards the car.
> >>
> >> Oh, don't doubt it. Metal in proximity is all it takes to bollix
> >> a signal. At the metal surface, E field is zero.
> >
> > Joe tried it.
> >
> >
> >
> >> I missed a lot of phone messages once, because I left my cell phone
> >> on a copper-clad counter. Felt a little silly when I was told no
> >> one could get in touch.

> Third has lived a sheltered life. ;)
>
> We built a couple of test stands for our fire and spark sensors used on
> cotton harvesters. The testers are built in a large die-cast aluminum
> stomp box with six screws in the lid.
>
> The Raspberry Pi inside has no trouble communicating to a nearby laptop
> via wifi.

Phil Hobbs forgets his EMI testing of closed boxes, or has somehow
never known that power/data/sensor cables were radiators of RF.
I'm guessing the Rpi didn't live alone in the dark without a few outside
world connections.

> For really effective shielding, you want an uncoated paint can, an old
> nickel-plated 70-mm film can, or a Danish Butter Cookies tin...

yep, all good solutions, if you aren't running power in and data or transducer
wiring out.

Re: Faraday bags and other scarey things

<e176ca62-10d9-ea95-55b6-3668d9fe0306@electrooptical.net>

  copy mid

https://www.novabbs.com/tech/article-flat.php?id=113395&group=sci.electronics.design#113395

  copy link   Newsgroups: sci.electronics.design
Path: i2pn2.org!i2pn.org!usenet.blueworldhosting.com!feed1.usenet.blueworldhosting.com!peer03.iad!feed-me.highwinds-media.com!news.highwinds-media.com!Xl.tags.giganews.com!local-1.nntp.ord.giganews.com!nntp.supernews.com!news.supernews.com.POSTED!not-for-mail
NNTP-Posting-Date: Sat, 31 Dec 2022 17:51:18 +0000
Subject: Re: Faraday bags and other scarey things
Newsgroups: sci.electronics.design
References: <0jbpqh9glc5nnc353uanjt33165o4reqao@4ax.com>
<toieom$2r24$1@dont-email.me> <43kpqhptg3bu556300bqemp473to7kvml2@4ax.com>
<0cmpqhhgqe6jv3kltna5broqnooj1rbd4i@4ax.com>
<sk2uqhtep8iojr9v1r45dobfp928m0cpmr@4ax.com>
<1ce0a10c-797b-48fa-a364-a8bf1eb581d6n@googlegroups.com>
<c6o0rht0r82pcn0htvaempt1pda0pqgsdl@4ax.com>
<1ab0342e-d340-a735-5e58-19d3648b8577@electrooptical.net>
<87282379-4f47-4abd-9849-ccc241aa3b6cn@googlegroups.com>
From: pcdhSpam...@electrooptical.net (Phil Hobbs)
Message-ID: <e176ca62-10d9-ea95-55b6-3668d9fe0306@electrooptical.net>
Date: Sat, 31 Dec 2022 12:51:17 -0500
User-Agent: Mozilla/5.0 (X11; Linux x86_64; rv:78.0) Gecko/20100101
Thunderbird/78.0
MIME-Version: 1.0
In-Reply-To: <87282379-4f47-4abd-9849-ccc241aa3b6cn@googlegroups.com>
Content-Type: text/plain; charset=UTF-8; format=flowed
Content-Transfer-Encoding: 7bit
Lines: 72
X-Trace: sv3-okmBXxZn7gdiR9RcB4Y5mflp1jLUMW5rjISpbXm41UgsOMSBqIWsb8R1H6oNZIIHHCY+wK0VhSHBfJY!jkQ1B22oAhclGIdOy6tdCNbo/FYKlhpa0qtoMsjwOKOZ5nyyTAH1vHCDA5gE5dvRJLrYWDSRxDTm!5gaQz3/sCywzWKBK1hZmlA==
X-Complaints-To: www.supernews.com/docs/abuse.html
X-DMCA-Complaints-To: www.supernews.com/docs/dmca.html
X-Abuse-and-DMCA-Info: Please be sure to forward a copy of ALL headers
X-Abuse-and-DMCA-Info: Otherwise we will be unable to process your complaint properly
X-Postfilter: 1.3.40
X-Received-Bytes: 4051
 by: Phil Hobbs - Sat, 31 Dec 2022 17:51 UTC

whit3rd wrote:
> On Saturday, December 31, 2022 at 8:47:16 AM UTC-8, Phil Hobbs wrote:
>> John Larkin wrote:
>>> On Fri, 30 Dec 2022 21:00:41 -0800 (PST), whit3rd
>>> <whi...@gmail.com> wrote:
>>>
>>>> On Friday, December 30, 2022 at 7:59:34 AM UTC-8, John Larkin
>>>> wrote:
>>>>> On Wed, 28 Dec 2022 19:09:10 -0500, Joe Gwinn
>>>>> <joeg...@comcast.net> wrote:
>
>>>>>> I bet keeping the fobs in an open top brass bowl would also
>>>>>> work, by being a shorted turn close to the fob's antenna loop.
>>>>>
>>>>> I doubt it.
>>>>>
>>>>> Put the fob in the bowl and walk towards the car.
>>>>
>>>> Oh, don't doubt it. Metal in proximity is all it takes to bollix
>>>> a signal. At the metal surface, E field is zero.
>>>
>>> Joe tried it.
>>>
>>>
>>>
>>>> I missed a lot of phone messages once, because I left my cell phone
>>>> on a copper-clad counter. Felt a little silly when I was told no
>>>> one could get in touch.
>
>> Third has lived a sheltered life. ;)
>>
>> We built a couple of test stands for our fire and spark sensors used on
>> cotton harvesters. The testers are built in a large die-cast aluminum
>> stomp box with six screws in the lid.
>>
>> The Raspberry Pi inside has no trouble communicating to a nearby laptop
>> via wifi.
>
> Phil Hobbs forgets his EMI testing of closed boxes, or has somehow
> never known that power/data/sensor cables were radiators of RF.
> I'm guessing the Rpi didn't live alone in the dark without a few outside
> world connections.
>
>> For really effective shielding, you want an uncoated paint can, an old
>> nickel-plated 70-mm film can, or a Danish Butter Cookies tin...
>
> yep, all good solutions, if you aren't running power in and data or transducer
> wiring out.

Of course, but entirely beside the point.

I might remind you that you said, "Metal in proximity is all it
takes...", which is spinach.

(And I used your nym because I was replying to JL's post, whereas you're
replying to mine. Saying "Phil Hobbs" this or that in a direct reply is
just too precious--shades of a middle school playground.)

Cheers

Phil Hobbs

--
Dr Philip C D Hobbs
Principal Consultant
ElectroOptical Innovations LLC / Hobbs ElectroOptics
Optics, Electro-optics, Photonics, Analog Electronics
Briarcliff Manor NY 10510

http://electrooptical.net
http://hobbs-eo.com

Re: Faraday bags and other scarey things

<9ps0rh5vd1bft12o9kc00dj67lt4q7cajh@4ax.com>

  copy mid

https://www.novabbs.com/tech/article-flat.php?id=113397&group=sci.electronics.design#113397

  copy link   Newsgroups: sci.electronics.design
Path: i2pn2.org!i2pn.org!usenet.blueworldhosting.com!feed1.usenet.blueworldhosting.com!peer01.iad!feed-me.highwinds-media.com!news.highwinds-media.com!feeder.usenetexpress.com!tr3.iad1.usenetexpress.com!69.80.99.23.MISMATCH!Xl.tags.giganews.com!local-2.nntp.ord.giganews.com!news.giganews.com.POSTED!not-for-mail
NNTP-Posting-Date: Sat, 31 Dec 2022 18:19:47 +0000
From: joegw...@comcast.net (Joe Gwinn)
Newsgroups: sci.electronics.design
Subject: Re: Faraday bags and other scarey things
Date: Sat, 31 Dec 2022 13:19:46 -0500
Message-ID: <9ps0rh5vd1bft12o9kc00dj67lt4q7cajh@4ax.com>
References: <0jbpqh9glc5nnc353uanjt33165o4reqao@4ax.com> <toieom$2r24$1@dont-email.me> <43kpqhptg3bu556300bqemp473to7kvml2@4ax.com> <0cmpqhhgqe6jv3kltna5broqnooj1rbd4i@4ax.com> <sk2uqhtep8iojr9v1r45dobfp928m0cpmr@4ax.com> <cucuqh19jgbdvclqam5m2le2227ve4gcr6@4ax.com> <27a4a011-c5c5-4a64-9ed1-2ae880b94602n@googlegroups.com> <lnmuqhtc3fa8qc2v3e8h5ik0qhhm7uh6kf@4ax.com> <6c3304c5-b054-471b-8fe2-2eac853219f5n@googlegroups.com>
User-Agent: ForteAgent/8.00.32.1272
MIME-Version: 1.0
Content-Type: text/plain; charset=us-ascii
Content-Transfer-Encoding: 7bit
Lines: 62
X-Usenet-Provider: http://www.giganews.com
X-Trace: sv3-DsqCVJZaWynsDziZDtsaii/Di1AJA5Sk4GLQtJSp2KyUPkIY39ukBi/HGiI7PWrHarN+gH2YdMMZBD9!CkWAUGNDtgbBIkVlpfDZcorj/+7LbPNmZhA1ywqeO4HGEvD7kvLQL7y1gAaxxwMfvUnFazA=
X-Complaints-To: abuse@giganews.com
X-DMCA-Notifications: http://www.giganews.com/info/dmca.html
X-Abuse-and-DMCA-Info: Please be sure to forward a copy of ALL headers
X-Abuse-and-DMCA-Info: Otherwise we will be unable to process your complaint properly
X-Postfilter: 1.3.40
X-Received-Bytes: 4400
 by: Joe Gwinn - Sat, 31 Dec 2022 18:19 UTC

On Fri, 30 Dec 2022 17:20:13 -0800 (PST), Ricky
<gnuarm.deletethisbit@gmail.com> wrote:

>On Friday, December 30, 2022 at 4:49:47 PM UTC-5, Joe Gwinn wrote:
>> On Fri, 30 Dec 2022 11:56:22 -0800 (PST), Ricky
>> <gnuarm.del...@gmail.com> wrote:
>>
>> >On Friday, December 30, 2022 at 2:22:21 PM UTC-5, Joe Gwinn wrote:
>> >>
>> >> Longer term, I think that the entire class of man-in-the-middle
>> >> attacks on key fobs will be solved by moving to some kind of
>> >> ultra-wideband signal, where one can measure round-trip time to with
>> >> picosecond precision, using a crypto-key sequence that cannot be
>> >> deduced from received signals fast enough to matter, thus allowing
>> >> too-distant fobs to be ignored with sufficiently low leakage to make
>> >> this kind of attack unprofitable.
>> >
>> >Why would you need ps accuracy in measuring round trip time? The distance you are trying to discriminate is around 100 feet, so 200 ns which can be done with a 40 MHz oscillator. There is no need to know to the foot, how far away the key is. Just is it more than 100 feet or so.
>> The distance from my kitchen to where the cars are parked outside (and
>> thus accessible to would-be thieves) is well less than 100 feet.
>
>You are saying you need what distance? What timing. Clearly, you don't need fractions of a foot, so not ps.
>
>
>> And picoseconds are easy for UWB technology.
>
>Who cares? It's not needed.
>
>
>> There is actually a use case that requires knowing the range: For
>> remote-start (very useful in cold places), the range may well be
>> large, but this does not unlock the car, so one can require no more
>> than six feet for opening and two or three feet for driving away.
>
>Actually, my car automatically opens the car door when I approach. From the rear, the door opens when I am about at the rear of the car. From the front, it opens when I'm about 3 feet, and only opens a crack, or it would be in my way. Then as I walk past the door, it opens the rest of the way. Very handy when my hands are full.

Three feet, then walk past. Hmm. So, there _is_ a use case needing
to know the distance a foot or so.

How does the Tesla know that you are three feet away, no farther no
closer? And then walked past?

>So I guess they've solved this issue without UWB tech. The battery in the fob lasts around a year.

Even if that's true, one could also use UWB. Or BlueTooth, which is
at 2.54 GHz. UWB is lower power than BlueTooth.

And UWB prox detection chips are widely available.

..<https://www.inpixon.com/technology/standards/ultra-wideband>

..<https://rtloc.com/adhoc/>

What's important here is not need per se, it's the ability to provide
features that customers will pay for, or select your offering versus
something else.

Joe Gwinn

Re: Faraday bags and other scarey things

<rd01rhlg3obkqm2qu43orsblvj0okb2onv@4ax.com>

  copy mid

https://www.novabbs.com/tech/article-flat.php?id=113400&group=sci.electronics.design#113400

  copy link   Newsgroups: sci.electronics.design
Path: i2pn2.org!i2pn.org!usenet.blueworldhosting.com!feed1.usenet.blueworldhosting.com!peer03.iad!feed-me.highwinds-media.com!news.highwinds-media.com!feeder.usenetexpress.com!tr1.iad1.usenetexpress.com!69.80.99.23.MISMATCH!Xl.tags.giganews.com!local-2.nntp.ord.giganews.com!nntp.supernews.com!news.supernews.com.POSTED!not-for-mail
NNTP-Posting-Date: Sat, 31 Dec 2022 18:48:35 +0000
From: jlar...@highlandSNIPMEtechnology.com (John Larkin)
Newsgroups: sci.electronics.design
Subject: Re: Faraday bags and other scarey things
Date: Sat, 31 Dec 2022 10:48:36 -0800
Organization: Highland Tech
Reply-To: xx@yy.com
Message-ID: <rd01rhlg3obkqm2qu43orsblvj0okb2onv@4ax.com>
References: <0jbpqh9glc5nnc353uanjt33165o4reqao@4ax.com> <toieom$2r24$1@dont-email.me> <43kpqhptg3bu556300bqemp473to7kvml2@4ax.com> <0cmpqhhgqe6jv3kltna5broqnooj1rbd4i@4ax.com> <sk2uqhtep8iojr9v1r45dobfp928m0cpmr@4ax.com> <cucuqh19jgbdvclqam5m2le2227ve4gcr6@4ax.com> <oghuqhpsllr02lijmmun7vk6f8fjio4oo1@4ax.com> <801e0792-e22c-5dc7-a404-3a0d6eb2cb3d@electrooptical.net> <shn0rhlru6puka0f34slflo8icmuul3bgd@4ax.com> <b7q0rhpfie5i7vkce4fhq880l4030k8ug7@4ax.com>
X-Newsreader: Forte Agent 3.1/32.783
MIME-Version: 1.0
Content-Type: text/plain; charset=us-ascii
Content-Transfer-Encoding: 7bit
Lines: 34
X-Trace: sv3-w9a3DQxOiBjp0i9McdICdLmxrForUV2Bb7yhDyCk4zNhavjxEXUsXZRRa3OFvIRZ1IipHldel6l/1ll!J+nN7BvFHdmg0sDB8TgMrDAB0x5knYuMsWlrHntG8hAQPQ1Me9TYqYw7FEj47L5qGSbOulP4kh/9!IpYCmg==
X-Complaints-To: www.supernews.com/docs/abuse.html
X-DMCA-Complaints-To: www.supernews.com/docs/dmca.html
X-Abuse-and-DMCA-Info: Please be sure to forward a copy of ALL headers
X-Abuse-and-DMCA-Info: Otherwise we will be unable to process your complaint properly
X-Postfilter: 1.3.40
X-Received-Bytes: 2615
 by: John Larkin - Sat, 31 Dec 2022 18:48 UTC

On Sat, 31 Dec 2022 11:56:08 -0500, Joe Gwinn <joegwinn@comcast.net>
wrote:
>
>>I got a little lithium battery jump starter as a backup. USB
>>rechargeable. It has USB power out too. I don't have to mess with
>>jumper cables to start my car, or someone else's car.
>>
>>It claims 1000 amps, which is improbable, but it started my V6 Audi
>>when it was cold and had a bad battery.
>
>This sounds worthwhile. Modern cars are actually too big for their
>batteries, by a factor.
>
>What make and model?

I got this one.

https://www.amazon.com/dp/B015TKUPIC

It seems fine so far, but I haven't had it for long. It is a great
concept, a small portable jump starter and USB power supply.

>
>
>>I'm impressed at what great mechanisms are mechanical keys and locks.
>>And how much bad code is around.
>
>Yes. Bob Pease's comment was that his best programming language was
>solder.

Kids these days use the horrible plastic proto things. Wusses.

>
>Joe Gwinn

Re: Faraday bags and other scarey things

<c121rh5e8dt3ceqmuavsodcae8telv5n5g@4ax.com>

  copy mid

https://www.novabbs.com/tech/article-flat.php?id=113401&group=sci.electronics.design#113401

  copy link   Newsgroups: sci.electronics.design
Path: i2pn2.org!i2pn.org!usenet.blueworldhosting.com!feed1.usenet.blueworldhosting.com!peer03.iad!feed-me.highwinds-media.com!news.highwinds-media.com!Xl.tags.giganews.com!local-1.nntp.ord.giganews.com!nntp.supernews.com!news.supernews.com.POSTED!not-for-mail
NNTP-Posting-Date: Sat, 31 Dec 2022 19:08:55 +0000
From: jlar...@highlandSNIPMEtechnology.com (John Larkin)
Newsgroups: sci.electronics.design
Subject: Re: Faraday bags and other scarey things
Date: Sat, 31 Dec 2022 11:08:56 -0800
Organization: Highland Tech
Reply-To: xx@yy.com
Message-ID: <c121rh5e8dt3ceqmuavsodcae8telv5n5g@4ax.com>
References: <toieom$2r24$1@dont-email.me> <43kpqhptg3bu556300bqemp473to7kvml2@4ax.com> <0cmpqhhgqe6jv3kltna5broqnooj1rbd4i@4ax.com> <sk2uqhtep8iojr9v1r45dobfp928m0cpmr@4ax.com> <cucuqh19jgbdvclqam5m2le2227ve4gcr6@4ax.com> <oghuqhpsllr02lijmmun7vk6f8fjio4oo1@4ax.com> <801e0792-e22c-5dc7-a404-3a0d6eb2cb3d@electrooptical.net> <shn0rhlru6puka0f34slflo8icmuul3bgd@4ax.com> <b7q0rhpfie5i7vkce4fhq880l4030k8ug7@4ax.com> <d2a69c45-8384-5f69-cd90-1df94b6b1e2a@electrooptical.net>
X-Newsreader: Forte Agent 3.1/32.783
MIME-Version: 1.0
Content-Type: text/plain; charset=us-ascii
Content-Transfer-Encoding: 7bit
Lines: 55
X-Trace: sv3-EqbDlk4yp8YWD+j9wWYop3KkTPqDMxfMYmlUOeoJPHAHx040HzFmrnyWGM+8m/olD1yZ+GQ7n5Mpbkq!WhDpMtzt7+FV6e/EZT+2u0Q4+oVZf4thJ5UNfjLd285LNZx3GLaNPZlcilFKtDeg2T5f6W+4QeFG!lPzsjA==
X-Complaints-To: www.supernews.com/docs/abuse.html
X-DMCA-Complaints-To: www.supernews.com/docs/dmca.html
X-Abuse-and-DMCA-Info: Please be sure to forward a copy of ALL headers
X-Abuse-and-DMCA-Info: Otherwise we will be unable to process your complaint properly
X-Postfilter: 1.3.40
X-Received-Bytes: 3104
 by: John Larkin - Sat, 31 Dec 2022 19:08 UTC

On Sat, 31 Dec 2022 12:21:28 -0500, Phil Hobbs
<pcdhSpamMeSenseless@electrooptical.net> wrote:

>Joe Gwinn wrote:
>> On Sat, 31 Dec 2022 08:17:09 -0800, John Larkin
>> <jlarkin@highlandSNIPMEtechnology.com> wrote:
>>
>>> On Sat, 31 Dec 2022 10:51:35 -0500, Phil Hobbs
>>> <pcdhSpamMeSenseless@electrooptical.net> wrote:
>>><snip>
>
>>>>>
>>>>> Why not make people push a button on the fob to unlock the car?
>>>>>
>>>>
>>>> Yeah, talk about solving a non-problem.
>>>>
>>>> Cheers
>>>>
>>>> Phil Hobbs
>>>
>>> My RF key fob has a little (mechanical) button that, when pressed, a
>>> metal key pops out! That will open doors even if all involved
>>> batteries are dead.
>>
>> I have one of those too.
>>
>>
>>> I got a little lithium battery jump starter as a backup. USB
>>> rechargeable. It has USB power out too. I don't have to mess with
>>> jumper cables to start my car, or someone else's car.
>>>
>>> It claims 1000 amps, which is improbable, but it started my V6 Audi
>>> when it was cold and had a bad battery.
>>
>> This sounds worthwhile. Modern cars are actually too big for their
>> batteries, by a factor.
>>
>> What make and model?
>>
>
>The Project Farm guy (Todd) has done a couple of jump-box comparos that
>are worth watching (at 1.75x, usually).
>
>https://www.youtube.com/watch?v=VRFwEUr_Vjc
>https://www.youtube.com/watch?v=5zAgL2L4ILY
>
>Cheers
>
>Phil Hobbs

There seem to be 10 Chinese amps per SI amp.

The Bureau of Consumer Protection is a joke.

Re: Faraday bags and other scarey things

<5821rh9q7eqjoa98345ia5ldguedmrpg1h@4ax.com>

  copy mid

https://www.novabbs.com/tech/article-flat.php?id=113402&group=sci.electronics.design#113402

  copy link   Newsgroups: sci.electronics.design
Path: i2pn2.org!i2pn.org!usenet.blueworldhosting.com!feed1.usenet.blueworldhosting.com!peer03.iad!feed-me.highwinds-media.com!news.highwinds-media.com!Xl.tags.giganews.com!local-1.nntp.ord.giganews.com!nntp.supernews.com!news.supernews.com.POSTED!not-for-mail
NNTP-Posting-Date: Sat, 31 Dec 2022 19:12:00 +0000
From: jlar...@highlandSNIPMEtechnology.com (John Larkin)
Newsgroups: sci.electronics.design
Subject: Re: Faraday bags and other scarey things
Date: Sat, 31 Dec 2022 11:12:02 -0800
Organization: Highland Tech
Reply-To: xx@yy.com
Message-ID: <5821rh9q7eqjoa98345ia5ldguedmrpg1h@4ax.com>
References: <0jbpqh9glc5nnc353uanjt33165o4reqao@4ax.com> <toieom$2r24$1@dont-email.me> <43kpqhptg3bu556300bqemp473to7kvml2@4ax.com> <0cmpqhhgqe6jv3kltna5broqnooj1rbd4i@4ax.com> <sk2uqhtep8iojr9v1r45dobfp928m0cpmr@4ax.com> <1ce0a10c-797b-48fa-a364-a8bf1eb581d6n@googlegroups.com> <c6o0rht0r82pcn0htvaempt1pda0pqgsdl@4ax.com> <1ab0342e-d340-a735-5e58-19d3648b8577@electrooptical.net>
X-Newsreader: Forte Agent 3.1/32.783
MIME-Version: 1.0
Content-Type: text/plain; charset=us-ascii
Content-Transfer-Encoding: 7bit
Lines: 36
X-Trace: sv3-7XZd4v0i+F0WgfhqtTBVYygfsTTqDyc1cm+/gUTOdD5fpse5T8fLhJ2jrSWjSJQjddLH+m8z/Ca20/z!vjk9C2kQtrpRJX557wjMZtyq/B1K33yDi+UvWjyFcduafFKYlcFR0VGgj8GWKUoXXiJoufRNvoZT!P9NXIQ==
X-Complaints-To: www.supernews.com/docs/abuse.html
X-DMCA-Complaints-To: www.supernews.com/docs/dmca.html
X-Abuse-and-DMCA-Info: Please be sure to forward a copy of ALL headers
X-Abuse-and-DMCA-Info: Otherwise we will be unable to process your complaint properly
X-Postfilter: 1.3.40
X-Received-Bytes: 2599
 by: John Larkin - Sat, 31 Dec 2022 19:12 UTC

On Sat, 31 Dec 2022 11:47:04 -0500, Phil Hobbs
<pcdhSpamMeSenseless@electrooptical.net> wrote:

>John Larkin wrote:
>> On Fri, 30 Dec 2022 21:00:41 -0800 (PST), whit3rd
>> <whit3rd@gmail.com> wrote:
>>
>>> On Friday, December 30, 2022 at 7:59:34 AM UTC-8, John Larkin
>>> wrote:
>>>> On Wed, 28 Dec 2022 19:09:10 -0500, Joe Gwinn
>>>> <joeg...@comcast.net> wrote:
>>>
>>>>> I bet keeping the fobs in an open top brass bowl would also
>>>>> work, by being a shorted turn close to the fob's antenna loop.
>>>>
>>>> I doubt it.
>>>>
>>>> Put the fob in the bowl and walk towards the car.
>>>
>>> Oh, don't doubt it. Metal in proximity is all it takes to bollix
>>> a signal. At the metal surface, E field is zero.
>>
>> Joe tried it.
>>
>>
>>
>>> I missed a lot of phone messages once, because I left my cell phone
>>> on a copper-clad counter. Felt a little silly when I was told no
>>> one could get in touch.
>
>Third has lived a sheltered life. ;)
>

Von Braun said that one experiment is worth a thousand expert
opinions. He didn't quantify the ratio for amateur opinions.

Re: Faraday bags and other scarey things

<bd7d0200-14c8-dd19-2888-be2619afe4d2@electrooptical.net>

  copy mid

https://www.novabbs.com/tech/article-flat.php?id=113405&group=sci.electronics.design#113405

  copy link   Newsgroups: sci.electronics.design
Path: i2pn2.org!i2pn.org!usenet.blueworldhosting.com!feed1.usenet.blueworldhosting.com!peer02.iad!feed-me.highwinds-media.com!news.highwinds-media.com!Xl.tags.giganews.com!local-1.nntp.ord.giganews.com!nntp.supernews.com!news.supernews.com.POSTED!not-for-mail
NNTP-Posting-Date: Sat, 31 Dec 2022 20:03:41 +0000
Subject: Re: Faraday bags and other scarey things
Newsgroups: sci.electronics.design
References: <toieom$2r24$1@dont-email.me>
<43kpqhptg3bu556300bqemp473to7kvml2@4ax.com>
<0cmpqhhgqe6jv3kltna5broqnooj1rbd4i@4ax.com>
<sk2uqhtep8iojr9v1r45dobfp928m0cpmr@4ax.com>
<cucuqh19jgbdvclqam5m2le2227ve4gcr6@4ax.com>
<oghuqhpsllr02lijmmun7vk6f8fjio4oo1@4ax.com>
<801e0792-e22c-5dc7-a404-3a0d6eb2cb3d@electrooptical.net>
<shn0rhlru6puka0f34slflo8icmuul3bgd@4ax.com>
<b7q0rhpfie5i7vkce4fhq880l4030k8ug7@4ax.com>
<d2a69c45-8384-5f69-cd90-1df94b6b1e2a@electrooptical.net>
<c121rh5e8dt3ceqmuavsodcae8telv5n5g@4ax.com>
From: pcdhSpam...@electrooptical.net (Phil Hobbs)
Message-ID: <bd7d0200-14c8-dd19-2888-be2619afe4d2@electrooptical.net>
Date: Sat, 31 Dec 2022 15:03:40 -0500
User-Agent: Mozilla/5.0 (X11; Linux x86_64; rv:78.0) Gecko/20100101
Thunderbird/78.0
MIME-Version: 1.0
In-Reply-To: <c121rh5e8dt3ceqmuavsodcae8telv5n5g@4ax.com>
Content-Type: text/plain; charset=UTF-8; format=flowed
Content-Transfer-Encoding: 7bit
Lines: 73
X-Trace: sv3-0maDCGfKAUCVWYX/74jMDYisNbhdzzugHzt4jp8QnnGk20LdgrXhwOG+kGpQFYcf9CTlEFkJfnbi2L7!PjB0hw74gA/wklZ+enlb9CYL4YGFt1HQyi2GNElkIQlXMImlJLyt329s3rLw69gQSP0Hb+R5xodd!+VLfyDEnjWeizdgmJwENxUI=
X-Complaints-To: www.supernews.com/docs/abuse.html
X-DMCA-Complaints-To: www.supernews.com/docs/dmca.html
X-Abuse-and-DMCA-Info: Please be sure to forward a copy of ALL headers
X-Abuse-and-DMCA-Info: Otherwise we will be unable to process your complaint properly
X-Postfilter: 1.3.40
X-Received-Bytes: 3752
 by: Phil Hobbs - Sat, 31 Dec 2022 20:03 UTC

John Larkin wrote:
> On Sat, 31 Dec 2022 12:21:28 -0500, Phil Hobbs
> <pcdhSpamMeSenseless@electrooptical.net> wrote:
>
>> Joe Gwinn wrote:
>>> On Sat, 31 Dec 2022 08:17:09 -0800, John Larkin
>>> <jlarkin@highlandSNIPMEtechnology.com> wrote:
>>>
>>>> On Sat, 31 Dec 2022 10:51:35 -0500, Phil Hobbs
>>>> <pcdhSpamMeSenseless@electrooptical.net> wrote:
>>>> <snip>
>>
>>>>>>
>>>>>> Why not make people push a button on the fob to unlock the car?
>>>>>>
>>>>>
>>>>> Yeah, talk about solving a non-problem.
>>>>>
>>>>> Cheers
>>>>>
>>>>> Phil Hobbs
>>>>
>>>> My RF key fob has a little (mechanical) button that, when pressed, a
>>>> metal key pops out! That will open doors even if all involved
>>>> batteries are dead.
>>>
>>> I have one of those too.
>>>
>>>
>>>> I got a little lithium battery jump starter as a backup. USB
>>>> rechargeable. It has USB power out too. I don't have to mess with
>>>> jumper cables to start my car, or someone else's car.
>>>>
>>>> It claims 1000 amps, which is improbable, but it started my V6 Audi
>>>> when it was cold and had a bad battery.
>>>
>>> This sounds worthwhile. Modern cars are actually too big for their
>>> batteries, by a factor.
>>>
>>> What make and model?
>>>
>>
>> The Project Farm guy (Todd) has done a couple of jump-box comparos that
>> are worth watching (at 1.75x, usually).
>>
>> https://www.youtube.com/watch?v=VRFwEUr_Vjc
>> https://www.youtube.com/watch?v=5zAgL2L4ILY
>>
>> Cheers
>>
>> Phil Hobbs
>
> There seem to be 10 Chinese amps per SI amp.
>
> The Bureau of Consumer Protection is a joke.
>
Yup. Way worse than even Marketing Megapixels. The 'boost button' is
key--lots of chargers won't work on a completely dead battery.

Cheers

Phil Hobbs

--
Dr Philip C D Hobbs
Principal Consultant
ElectroOptical Innovations LLC / Hobbs ElectroOptics
Optics, Electro-optics, Photonics, Analog Electronics
Briarcliff Manor NY 10510

http://electrooptical.net
http://hobbs-eo.com

Re: Faraday bags and other scarey things

<27cd8c14-5998-4dc4-b593-0669c321e5b9n@googlegroups.com>

  copy mid

https://www.novabbs.com/tech/article-flat.php?id=113407&group=sci.electronics.design#113407

  copy link   Newsgroups: sci.electronics.design
X-Received: by 2002:ad4:5909:0:b0:4c7:343d:2a60 with SMTP id ez9-20020ad45909000000b004c7343d2a60mr1909380qvb.42.1672523642780;
Sat, 31 Dec 2022 13:54:02 -0800 (PST)
X-Received: by 2002:a37:a93:0:b0:6ff:812e:a82f with SMTP id
141-20020a370a93000000b006ff812ea82fmr1547403qkk.336.1672523642548; Sat, 31
Dec 2022 13:54:02 -0800 (PST)
Path: i2pn2.org!i2pn.org!usenet.blueworldhosting.com!feed1.usenet.blueworldhosting.com!peer03.iad!feed-me.highwinds-media.com!news.highwinds-media.com!news-out.google.com!nntp.google.com!postnews.google.com!google-groups.googlegroups.com!not-for-mail
Newsgroups: sci.electronics.design
Date: Sat, 31 Dec 2022 13:54:02 -0800 (PST)
In-Reply-To: <bd7d0200-14c8-dd19-2888-be2619afe4d2@electrooptical.net>
Injection-Info: google-groups.googlegroups.com; posting-host=65.207.89.54; posting-account=I-_H_woAAAA9zzro6crtEpUAyIvzd19b
NNTP-Posting-Host: 65.207.89.54
References: <toieom$2r24$1@dont-email.me> <43kpqhptg3bu556300bqemp473to7kvml2@4ax.com>
<0cmpqhhgqe6jv3kltna5broqnooj1rbd4i@4ax.com> <sk2uqhtep8iojr9v1r45dobfp928m0cpmr@4ax.com>
<cucuqh19jgbdvclqam5m2le2227ve4gcr6@4ax.com> <oghuqhpsllr02lijmmun7vk6f8fjio4oo1@4ax.com>
<801e0792-e22c-5dc7-a404-3a0d6eb2cb3d@electrooptical.net> <shn0rhlru6puka0f34slflo8icmuul3bgd@4ax.com>
<b7q0rhpfie5i7vkce4fhq880l4030k8ug7@4ax.com> <d2a69c45-8384-5f69-cd90-1df94b6b1e2a@electrooptical.net>
<c121rh5e8dt3ceqmuavsodcae8telv5n5g@4ax.com> <bd7d0200-14c8-dd19-2888-be2619afe4d2@electrooptical.net>
User-Agent: G2/1.0
MIME-Version: 1.0
Message-ID: <27cd8c14-5998-4dc4-b593-0669c321e5b9n@googlegroups.com>
Subject: Re: Faraday bags and other scarey things
From: gnuarm.d...@gmail.com (Ricky)
Injection-Date: Sat, 31 Dec 2022 21:54:02 +0000
Content-Type: text/plain; charset="UTF-8"
X-Received-Bytes: 3801
 by: Ricky - Sat, 31 Dec 2022 21:54 UTC

On Saturday, December 31, 2022 at 3:03:52 PM UTC-5, Phil Hobbs wrote:
> John Larkin wrote:
> > On Sat, 31 Dec 2022 12:21:28 -0500, Phil Hobbs
> > <pcdhSpamM...@electrooptical.net> wrote:
> >
> >> Joe Gwinn wrote:
> >>> On Sat, 31 Dec 2022 08:17:09 -0800, John Larkin
> >>> <jla...@highlandSNIPMEtechnology.com> wrote:
> >>>
> >>>> On Sat, 31 Dec 2022 10:51:35 -0500, Phil Hobbs
> >>>> <pcdhSpamM...@electrooptical.net> wrote:
> >>>> <snip>
> >>
> >>>>>>
> >>>>>> Why not make people push a button on the fob to unlock the car?
> >>>>>>
> >>>>>
> >>>>> Yeah, talk about solving a non-problem.
> >>>>>
> >>>>> Cheers
> >>>>>
> >>>>> Phil Hobbs
> >>>>
> >>>> My RF key fob has a little (mechanical) button that, when pressed, a
> >>>> metal key pops out! That will open doors even if all involved
> >>>> batteries are dead.
> >>>
> >>> I have one of those too.
> >>>
> >>>
> >>>> I got a little lithium battery jump starter as a backup. USB
> >>>> rechargeable. It has USB power out too. I don't have to mess with
> >>>> jumper cables to start my car, or someone else's car.
> >>>>
> >>>> It claims 1000 amps, which is improbable, but it started my V6 Audi
> >>>> when it was cold and had a bad battery.
> >>>
> >>> This sounds worthwhile. Modern cars are actually too big for their
> >>> batteries, by a factor.
> >>>
> >>> What make and model?
> >>>
> >>
> >> The Project Farm guy (Todd) has done a couple of jump-box comparos that
> >> are worth watching (at 1.75x, usually).
> >>
> >> https://www.youtube.com/watch?v=VRFwEUr_Vjc
> >> https://www.youtube.com/watch?v=5zAgL2L4ILY
> >>
> >> Cheers
> >>
> >> Phil Hobbs
> >
> > There seem to be 10 Chinese amps per SI amp.
> >
> > The Bureau of Consumer Protection is a joke.
> >
> Yup. Way worse than even Marketing Megapixels. The 'boost button' is
> key--lots of chargers won't work on a completely dead battery.

I never have any trouble charging my battery.

--

Rick C.

-+- Get 1,000 miles of free Supercharging
-+- Tesla referral code - https://ts.la/richard11209

Re: Faraday bags and other scarey things

<vbd1rh1talj91b8ouk0dmlums632st1k3c@4ax.com>

  copy mid

https://www.novabbs.com/tech/article-flat.php?id=113410&group=sci.electronics.design#113410

  copy link   Newsgroups: sci.electronics.design
Path: i2pn2.org!i2pn.org!usenet.blueworldhosting.com!feed1.usenet.blueworldhosting.com!peer03.iad!feed-me.highwinds-media.com!news.highwinds-media.com!feeder.usenetexpress.com!tr3.iad1.usenetexpress.com!69.80.99.18.MISMATCH!border-1.nntp.ord.giganews.com!nntp.giganews.com!Xl.tags.giganews.com!local-2.nntp.ord.giganews.com!nntp.supernews.com!news.supernews.com.POSTED!not-for-mail
NNTP-Posting-Date: Sat, 31 Dec 2022 22:22:24 +0000
From: jlar...@highlandSNIPMEtechnology.com (John Larkin)
Newsgroups: sci.electronics.design
Subject: Re: Faraday bags and other scarey things
Date: Sat, 31 Dec 2022 14:22:25 -0800
Organization: Highland Tech
Reply-To: xx@yy.com
Message-ID: <vbd1rh1talj91b8ouk0dmlums632st1k3c@4ax.com>
References: <sk2uqhtep8iojr9v1r45dobfp928m0cpmr@4ax.com> <cucuqh19jgbdvclqam5m2le2227ve4gcr6@4ax.com> <oghuqhpsllr02lijmmun7vk6f8fjio4oo1@4ax.com> <801e0792-e22c-5dc7-a404-3a0d6eb2cb3d@electrooptical.net> <shn0rhlru6puka0f34slflo8icmuul3bgd@4ax.com> <b7q0rhpfie5i7vkce4fhq880l4030k8ug7@4ax.com> <d2a69c45-8384-5f69-cd90-1df94b6b1e2a@electrooptical.net> <c121rh5e8dt3ceqmuavsodcae8telv5n5g@4ax.com> <bd7d0200-14c8-dd19-2888-be2619afe4d2@electrooptical.net>
X-Newsreader: Forte Agent 3.1/32.783
MIME-Version: 1.0
Content-Type: text/plain; charset=us-ascii
Content-Transfer-Encoding: 7bit
Lines: 68
X-Trace: sv3-eKwPcMmZJE0q488VTuiJi539ZrB3yUeAzwRQz2iPWuhquH14XrIae9YeFFwLUKILYbDvIoTKaTI8Vr5!xi3u8KdCDdbI0be64+XKpYn1J2teAHgCmht47RKz8+EEuyiG60jFnSkIUswiY687S97DkJIuBifD!grCVwg==
X-Complaints-To: www.supernews.com/docs/abuse.html
X-DMCA-Complaints-To: www.supernews.com/docs/dmca.html
X-Abuse-and-DMCA-Info: Please be sure to forward a copy of ALL headers
X-Abuse-and-DMCA-Info: Otherwise we will be unable to process your complaint properly
X-Postfilter: 1.3.40
X-Received-Bytes: 3772
 by: John Larkin - Sat, 31 Dec 2022 22:22 UTC

On Sat, 31 Dec 2022 15:03:40 -0500, Phil Hobbs
<pcdhSpamMeSenseless@electrooptical.net> wrote:

>John Larkin wrote:
>> On Sat, 31 Dec 2022 12:21:28 -0500, Phil Hobbs
>> <pcdhSpamMeSenseless@electrooptical.net> wrote:
>>
>>> Joe Gwinn wrote:
>>>> On Sat, 31 Dec 2022 08:17:09 -0800, John Larkin
>>>> <jlarkin@highlandSNIPMEtechnology.com> wrote:
>>>>
>>>>> On Sat, 31 Dec 2022 10:51:35 -0500, Phil Hobbs
>>>>> <pcdhSpamMeSenseless@electrooptical.net> wrote:
>>>>> <snip>
>>>
>>>>>>>
>>>>>>> Why not make people push a button on the fob to unlock the car?
>>>>>>>
>>>>>>
>>>>>> Yeah, talk about solving a non-problem.
>>>>>>
>>>>>> Cheers
>>>>>>
>>>>>> Phil Hobbs
>>>>>
>>>>> My RF key fob has a little (mechanical) button that, when pressed, a
>>>>> metal key pops out! That will open doors even if all involved
>>>>> batteries are dead.
>>>>
>>>> I have one of those too.
>>>>
>>>>
>>>>> I got a little lithium battery jump starter as a backup. USB
>>>>> rechargeable. It has USB power out too. I don't have to mess with
>>>>> jumper cables to start my car, or someone else's car.
>>>>>
>>>>> It claims 1000 amps, which is improbable, but it started my V6 Audi
>>>>> when it was cold and had a bad battery.
>>>>
>>>> This sounds worthwhile. Modern cars are actually too big for their
>>>> batteries, by a factor.
>>>>
>>>> What make and model?
>>>>
>>>
>>> The Project Farm guy (Todd) has done a couple of jump-box comparos that
>>> are worth watching (at 1.75x, usually).
>>>
>>> https://www.youtube.com/watch?v=VRFwEUr_Vjc
>>> https://www.youtube.com/watch?v=5zAgL2L4ILY
>>>
>>> Cheers
>>>
>>> Phil Hobbs
>>
>> There seem to be 10 Chinese amps per SI amp.
>>
>> The Bureau of Consumer Protection is a joke.
>>
>Yup. Way worse than even Marketing Megapixels. The 'boost button' is
>key--lots of chargers won't work on a completely dead battery.

The car parts people sell those kinds of chargers, because it leads to
more battery sales.

I keep a bench power supply around the house. It will charge a totally
dead battery.


tech / sci.electronics.design / Re: Faraday bags and other scarey things

Pages:12
server_pubkey.txt

rocksolid light 0.9.81
clearnet tor